Pharmocology

Ace your homework & exams now with Quizwiz!

The nurse administers lipids (fat emulsion solution) during TPN as ordered based on the understanding that this type of solution: A. provides essential fatty acids B. provides extra carbohydrates C. promotes effective metabolism of glucose D. maintains normal body weight

A

The nurse is assessing a patient receiving metformin. Which of the following assessments is a priority? A. Renal function B. WBC cell count C. Deep tendon reflexes D. Hemoglobin

A

The nurse is starting atorvastatin (Lipitor) for a patient with hypercholesterolemia. For which of the following conditions is atorvastatin (Lipitor) contraindicated? A. serious liver disorder B. renal dysfunction C. stroke D. biliary obstruction

A

The drug of choice for Type 2 diabetics is metformin. Metformin: A. Decreases glycogenolysis by the liver B. Increases the release of insulin from beta cells C. Increases intestinal uptake of glucose D. Prevents weight gain associated with hyperglycemia

A

BUN

.5- 20 mg (high bun means blood is concentrated)

Transdermal delivery

0.1 mg/24 hours 0.2 mg/24 hours 0.3 mg/24 hours

Normal creatinine

0.6- 1.2 (younger than 50 or 60 with all limbs)

Digoxin level

0.8 ng/mL - 1.0 ng/dL

Normal creatinine clearance

100

Beta blockers

2 main beta receptors: beta 1 (in cardiac/vascular tissue) and beta 2 (in lungs and smooth muscles) Selective: atenolol, metoprolol Primarily block beta 1 Nonselective: propanolol (Inderal) Effect beta 1 and beta 2 receptors This category of Beta blockers also impact smooth muscles

LDL Levels

< 100 Optimal 100 - 129 Near optimal/above optimal 130 - 159 Borderline high 160 - 189 High > 190 Very high

Cholesterol

< 200 Desirable 200 - 239 Borderline high > 240 High

HDL levels

< 40 Low (Bad thing!) > 60 High (Good thing!)

A The nurse is following a patient on metformin. Which of the following laboratory parameters would create the greatest concern? A. Elevated creatinine B. Anemia C. Decreased platelets D. Increased iron

A

A patient with newly diagnosed diabetes mellitus is instructed by the physician to obtain glucagon for emergency home use. The patient asks a home care nurse about the purpose of the medication. The nurse instructs the patient that the purpose of the medication is to treat which of the following? A. Hypoglycemia from insulin overdose B. Hyperglycemia from insufficient insulin C. Lipoatrophy from insulin injections D. Liperhypertrophy from inadequate insulin absorption

A

Because of its action on various body systems, the patient taking a thiazide or loop diuretic may also need to receive: Potassium supplements Calcium supplements Magnesium supplements Phosphates supplements

A

The nurse has been caring for a patient who has been taking antibiotics for 3 weeks. Upon assessing the patient, the nurse notices the individual has developed oral thrush. Which of the following describes the etiology of the thrush? A. Suprainfection B. Antibiotic resistance C. Nosocomial infection D. Community-acquired infection

A Oral thrush is a manifestation of a suprainfection. The development of thrush is not a symptom of antibiotic resistance. The development of thrush is not a nosocomial infection. Oral thrush typically is not a community-acquired infection.

When teaching a patient about the proper application of timolol eye drops, the nurse will include which instruction? A. Apply the drops into the conjunctival sac instead of directly onto the eye. B. Apply the drops directly to the cornea for the best effectiveness. C. Blot the eye with a tissue immediately after application of the drops. D. Gently touch the tip of the dropper to the eye surface before administering the drop.

A All ophthalmic drugs should be administered in the conjunctival sac. Avoid touching the eye with the tip of the dropper to prevent contamination of the product. Gently use a tissue to remove excess eye medication—do not blot the eye after giving the medication.

The nurse is caring for a patient admitted with syncope and sinus bradycardia. The nurse is passing morning medications and prepares to administer medication for glaucoma. Which category of drugs for glaucoma could exacerbate the patient's heart problems? A. Beta-adrenergic blockers B. Prostaglandin analogs C. Alpha2-adrenergic agonists D. Anticholinergic agonists

A Blockade of cardiac beta1 receptors can produce bradycardia and AV heart block, as well as depress cardiac function. Beta-adrenergic blockers would be contraindicated for patients with existing AV heart block, sinus bradycardia, or cardiogenic shock. Prostaglandin analogs do not adversely affect the heart. Alpha2-adrenergic agonists and anticholinergic agonists do not exacerbate heart problems.

The nurse is serving as a preceptor for a graduate nurse on a medical-surgical unit. The nurse, who is discussing the mechanisms of action of antibiotics, currently is talking about the mechanism of action of penicillins (PCNs). Which statement by the graduate nurse best demonstrates understanding? "Penicillin works by A. weakening the bacterial cell wall, which then ruptures and dies." B. inhibiting protein synthesis and suppressing bacterial growth." C. disrupting bacterial protein synthesis, destroying the bacterial wall." D. disrupting biochemical reactions, which results in cellular lysis."

A PCNs weaken the cell wall, causing bacteria to take up excessive amounts of water and subsequently rupture. PCNs do not inhibit protein synthesis, disrupt bacterial protein synthesis, or disrupt biochemical reactions.

A patient comes to the clinic for a follow-up appointment after using a glucocorticoid ointment for eczema for 3 weeks. Because the patient is pleased with the results, she states that she would like to continue treatment with an oral glucocorticoid. What response from the nurse is most appropriate? A. "Oral glucocorticoids are only indicated if topical glucocorticoids have been ineffective." B. "Oral glucocorticoids will have fewer side effects, so your suggestion is good." C. "You make an excellent point! Topical glucocorticoids can cause skin atrophy, whereas oral agents don't have the same effect." D. "Oral glucocorticoids are more convenient to use, so I agree with your suggestion."

A The nurse should tell the patient that there is no basis for administration of oral glucocorticoids, because the topical agent has been effective. Oral glucocorticoids have more serious side effects (eg, adrenal suppression), and there is no basis for their use. Whether applied topically or taken orally, glucocorticoids can still lead to skin atrophy. Convenience is not an adequate reason for switching to oral agents when the topical agents have been effective.

The nurse is providing education to a 21-year-old construction worker who is being treated for severe acne. The prescriber orders isotretinoin (Accutane) as treatment for the acne. Which instruction is most pertinent for the nurse to provide? A. "Wear protective clothing when in the sun." B. "Use abrasive soap to aid desquamation." C. "Discontinue use of the drug if flaking of the skin occurs." D. "Wear sunglasses as protection against photophobia."

A The patient should be instructed to wear protective clothing when in the sun, because isotretinoin may make the skin sensitive to ultraviolet light. An abrasive soap is not indicated with this treatment and may cause further irritation. Isotretinoin is expected to cause some flaking; it should not be discontinued. Photophobia is not an adverse effect of isotretinoin.

A patient admitted for an acute outbreak of herpes-zoster virus is receiving valacyclovir (Valtrex). The nurse provides patient education about valacyclovir (Valtrex). What comment by the patient indicates a need for further teaching? A. "Valtrex will help reduce the duration of my pain." B. "Valtrex will help reduce the duration of postherpetic neuralgia." C. "Valtrex will prevent recurrent infection." D. "Valtrex may be taken with or without food."

A Valacyclovir will not prevent recurrent infection; this statement indicates that further teaching is necessary. Valacyclovir does help reduce the duration of pain and symptoms of postherpetic neuralgia; no further teaching is necessary. Valacyclovir may be taken with or without food; no further teaching is necessary.

The nurse is caring for a patient receiving gentamicin (Garamycin) intravenously (IV). What statement by the patient would most concern the nurse? "I am experiencing A. 30 minutes after the IV infusion is complete B. 1 hour after the IV infusion is complete C. 1 hour prior to administration of the IV infusion D. A peak level is not indicated.

A When divided daily doses are used, blood samples for measurement of peak levels are drawn 1 hour after IM injection and 30 minutes after completion of an IV infusion. This medication is administered IV, so blood draws must follow 30 after infusion to obtain peak levels. Measurement of peak levels is unnecessary only when a single daily does is used.

Prior to administering a laxative to a client - what assessments are essential?

A contraindication to laxatives is an acute abdomen. Administering a laxative in this situation may cause perforation. Perform a GI assessment: look for abdominal pain, nausea and cramps. Also signs of peritoneal irritation (like rebound tenderness)

Peptic ulcer disease (PUD)

A general group of disorders - eroded gut wall

. A nurse is assigned to administer regular insulin to a patient with type 1 diabetes. When should the nurse administer the insulin? A. 15 minutes prior to the meal. B. 30 - 60 minutes prior to the meal C. once at bedtime via the sc route. D. 30 minutes after eating a meal

A is correct, the glucose of 482 is above the scale listed

When teaching a client who is to take nitroglycerin tablets, the nurse should include the importance of A. limiting the number of tablets to three per day. B. discontinuing the medication if a headache develops. C. making certain the medication is stored in a dark container. D. increasing the number of tablets is dizziness or hypertension occurs.

A is incorrect because the patient can take one tablet every 5 minutes for three doses. B is incorrect because often this medication will give a patient a headache due to vasodilation. D is incorrect because nitroglycerin has specific instructions and a patient should not be told to titrate the medication to blood pressure because the medication is indicated for angina.

When teaching a client who is to take nitroglycerin tablets, the nurse should include the importance of A. limiting the number of tablets to three per day. B. discontinuing the medication if a headache develops. C. making certain the medication is stored in a dark container. D. increasing the number of tablets is dizziness or hypertension occurs.

A( is incorrect because the patient can take one tablet every 5 minutes for three doses. B is incorrect because often this medication will give a patient a headache due to vasodilation. D is incorrect because nitroglycerin has specific instructions and a patient should not be told to titrate the medication to blood pressure because the medication is indicated for angina.)

A patient is being treated with a cholestyramine (Questran). Which of the following adverse effects should the nurse monitor for? Select all that apply A. hard dry stools B. painful stools C. mouth dryness D. urinary hesitancy E. constipation

A, B, E

A client is taking furosemide (Lasix). The nurse monitors for which of the following signs and symptoms of hypokalemia? Select all that apply A. Tall T waves B. Diaphoresis C. Deep tendon hyporeflexia D. Musculoskeletal weakness E. Hypoactive to absent bowel sounds F. Dorsalis pedis pulses + 2 bilaterally

A, D

The nurse is teaching a class on drugs for the skin to a group of nursing students. The nurse asks the class, "For which purposes may topical glucocorticoids be used?" The students would be correct to respond that glucocorticoids may be used for treatment of which of the following? (Select all that apply.) A. Insect bites B. Psoriasis C. Minor burns D. Mild skin infection

A,B,C Topical glucocorticoids are used to relieve inflammation and itching associated with a variety of dermatological disorders, such as insect bites, psoriasis, minor burns, seborrheic dermatitis, eczema, and pemphigus. Glucocorticoids are not indicated for skin infections and can increase the risk of local infection if used.

79. Which information should be included in a teaching plan for patients taking oral hypoglycemic drugs? (Select all that apply.) A. Limit your alcohol consumption B. Report symptoms of anorexia and fatigue C. Take your medications only as needed D. Notify your physician if your blood glucose rises above the level setl

A,B,D

A 16-year-old patient is diagnosed with severe nodulocystic acne vulgaris and is to receive treatment with isotretinoin (Accutane). What patient education about medication side effects is appropriate for the nurse to provide? (Select all that apply.) A. "You should wear protective clothing or sunscreen when outside." B. "This medication may cause nosebleeds." C. "You may experience an increase in hair growth." D. "You may experience dryness or itching of the skin and nose."

A,B,D During treatment with isotretinoin, the patient's skin may become sensitized to ultraviolet light; therefore, instructions to use sunscreen and wear protective clothing while outside are important. The nurse also should advise the patient that side effects associated with isotretinoin are nosebleeds; inflammation of the lips and eyes; dryness or itching of the skin, nose, and mouth; and pain, tenderness or stiffness in muscles, bones, and joints. A less common reaction is hair loss, not increased hair growth. Isotretinoin does not cause sores in the mouth

FDA Categories

A- Insulin & Thyroid B- best C- caution- most drugs D- don't x- no

The physician orders insulin lispro (Humalog) 10 units for the client. When will the nurse administer this medication? A. Fifteen minutes before meals B. When the meal trays arrive on the floor C. When the client is eating D. Thirty minutes before meals

A. Rationale: The onset of action for insulin lispro (Humalog) is 10 to 15 minutes so it must be given when the client is eating to prevent hypoglycemia. The onset of action for insulin lispro (Humalog) is 10 to 15 minutes so it must be given when the client is eating, not fifteen minutes before meals, to prevent hypoglycemia. The onset of action for insulin lispro (Humalog) is 10 to 15 minutes so it must be given when the client is eating, not when meal trays arrive on the floor, to prevent hypoglycemia. The onset of action for insulin lispro (Humalog) is 10 to 15 minutes so it must be given when the client is eating, not thirty minutes before meals, to prevent hypoglycemia

N-acetylcysteine (mucomyst)

Acetaminophen antidote

Drug distribution results in which of the following? A. Binding of drugs to plasma proteins B. Reduced cardiac output with increased tissue binding C. Movement of drugs from the body through the bile D. Saturation of the drug into tissues with poor blood supply

A. Binding of drugs to plasma proteins

The physician has prescribed sertraline (Zoloft) for the client with depression. The client has stopped taking the medication. The nurse incorrectly uses the diagnosis of "noncompliance." What is essential for the nurse to assess prior to using this nursing diagnosis? Select all that apply. A. Did the client understand why the medication had been prescribed? B. Are side effects causing the client to refuse the medication? C. Do cultural or religious issues have an impact on taking the medication? D. Has the client made an informed decision not to take the medication? E. Is the noncompliance related to a lack of finances?

A. Did the client understand why the medication had been prescribed? B. Are side effects causing the client to refuse the medication? D. Has the client made an informed decision not to take the medication? E. Is the noncompliance related to a lack of finances? Rationale: A lack of understanding of the reason the medication was prescribed, the occurrence of side effects, cultural or religious issues, and a lack of finances can all contribute to noncompliance with medications. Noncompliance assumes that the client has been properly educated about the medication and has made an informed decision not to take it. The nursing diagnosis of noncompliance would not be appropriate in this case.

Calcium channel blockers: Phenylakylamines

Verapamil and diltiazem Action: non selective = affect arterioles and heart Inhibit influx of calcium Adverse effects: Constipation Bradycardia Interactions: Digoxin and beta blockers

5. A nurse is preparing to administer epinephrine to a patient who is having a severe allergic reaction. Which route of administration would provide the fastest and most complete absorption of epinephrine? A. Intravenous B. Intramuscular C. Subcutaenous D. Oral

A. Intravenous

The nurse is administering medications to an elderly client. Which laboratory tests are important for the nurse to assess prior to the administration of medication? A. Kidney and liver function tests B. Complete blood count (CBC) and electrolytes C. Lipid panel and thyroid function tests D. Arterial blood gases (ABGs) and basic metabolic panel

A. Kidney and liver function tests Rationale: Renal and hepatic function tests are essential for many clients, particularly older clients and those who are critically ill, as these will be used to determine the proper drug dosage. Complete blood count (CBC) and electrolytes will not help to determine the proper drug dosage. Lipid panel and thyroid function tests will not help to determine the proper drug dosage. Arterial blood gases (ABGs) and a basic metabolic panel will not help to determine the proper drug dosage.

A potentially life-threatening adverse response to certain medications is angioedema. Which of the following statements is true about this adverse response? A. Swelling of the tongue or hoarseness are the most common symptoms. B. It appears to be related to the decrease in aldosterone production. C. Presence of a dry, hacky cough indicates a high risk for this adverse response. D. Because it takes time to build up a blood level, it occurs after being on the drug for about 1 week.

A. Swelling of the tongue or hoarseness are the most common symptoms.

IIb/IIIa antagonists

Abciximab (Reopro), Tirofiban (Aggrastat) Mechanism of Action: causes irreversible blockade of ADP which prevents aggregation Indication: short term medications Adjunct to percutaneous interventions Also known as: stent, angioplasty Acute coronary syndrome

Creatinine

Accumulation of muscle breakdown.

Isotretionin

Accutane Mechanism of action: Decreases sebum (nutrient for s. pyogenes) production Decreases sebaceous gland size, inflammation and keratinization Adverse effects: Frequent and dose related Teratogenicity Depression Dry mouth and eyes Muscle and joint pain Alopecia Hypertriglyceridemia - reversible with discontinuation of drug Photosensitivity - patient needs sunscreens Category X FDA mandates that the patient register in online registry Liver function and serum lipids at baseline, 4 and 8 weeks Assure 2 negative pregnancy tests prior to initiation Patients must commit to effective birth control

Mucolytics

Acetylcysteine (Mucomyst) Therapeutic uses: Chronic bronchitis Antidote for tylenol overdose

Immunomodulators

Adalimumab (Humira) Inflixamab (Remicade) Mechanism of action: Tumor necrosis factor inhibition Adverse effects: Serious infection Malignancy Nursing Care: Assess: monitor CBC, creatinine and LFT's Obtain negative tuberculosis test prior to starting, hepatitis C Implementation: Infliximab infused over 2 hours Adalimumab administered subcutaneously Instruct patient to avoid live vaccinations Instruct patient to report severe infections Instruct patient to report dizziness, chills, dyspnea, seizures or fatigue or rash immediately

Immunomodulators

Adalimumab (Humira) - RA. TNF blockade Inflixamab (Remicade) - RA. TNF blockade trastuzumab (Herceptin) - Cancer alemtuzumab (Campath) - Cancer Adverse effects: Serious infection Malignancy Instruct patient to avoid live vaccinations: Flu Mist - nasal, Herpes Zoster - Zostavax (Shingles), MMR (measles, mumps, rubella), Rotavirus, Varicella Instruct patient to report severe infections Instruct patient to report dizziness, chills, dyspnea, seizures or fatigue or rash immediately Anaphylaxis Hypersensitivity

Administration of ________________ results in a viscid and very sticky gel that adheres to the ulcer crater, creating a barrier to back-diffusion of hydrogen ions, pepsin, and bile salts.

Administration of __sucralfate________________ results in a viscid and very sticky gel that adheres to the ulcer crater, creating a barrier to back-diffusion of hydrogen ions, pepsin, and bile salts. This is great because it coats the ulcer bed for so long - however, it also coats the possible absorption sites of medications. So be careful when you administer drugs - you may be preventing the medication from being absorbed.

Doxorubicin Antitumor Antibiotics

Adriamycin, other -cin drugs Mechanism of action: Non specific Inserts itself into DNA distorting it Acute and delayed cardiotoxicity - limits doses Bleomycin (Blenoxane) Mechanism of action: G phase specific Binds to DNA and disrupts Pneumonitis and pulmonary fibrosis - 10%!

Sulfonamides

Adverse effects: "Mild" reactions are more common rash, drug fever, photosensitivity Hypersensitivity reactions Blood dyscrasias - generic predisposition Steven's Johnson Syndrome Kernicteris Renal damage: Low solubility Crystal form Allergies - beware of related drugs (thiazide diuretics, loop diuretics, sulfonyurea oral diabetic agents)

When administering spironalactone (Aldactone) it is important to monitor for what electrolyte abnormality? What are the signs and symptoms of this condition?

Although rare, the most serious adverse effect of spironalactone is symptomatic hyperkalemia Symptoms of hyperkalemia may include paresthesias, muscular weakness, fatigue, flaccid paralysis of the extremities, bradycardia, shock and ECG abnormalities. Spironalactone has been shown to have some sexual side effects (such as men developing larger than normal mammary glands)

Antacids

Aluminum hydroxide (AlternaGEL) Calcium carbonate (Tums) Magnesium hydroxide (Maalox) Mechanism of action: Alkaline compounds that neutralize stomach acid Reduce pepsin activity May also enhance mucosal protection - by stimulating production of prostaglandins Therapeutic use: PUD and GERD

Amphotericin

Ambisome - intravenous Broad spectrum antifungal Mechanism of action: binds to cell membrane - cell will explode Therapeutic use: for systemic mycoses ( used when fungus is in blood stream) Incredible risk of harm - prolonged treatment Adverse effects: Infusion issues Nephrotoxicity

AntiAntibiotics For H. Pylori

Amoxicillin Clarithromycin Tetracycline Prescribers will choose two different antibiotics - to prevent bacterial resistance Bismuth subsalicylate (Pepto Bismol):

Drug schedule 3

Anabolic steriods, codeine, hydrocodone

A nurse is caring for a patient receiving bolus enteral feedings. After finishing the feedings, the patient asks to be positioned for sleep. The nurse understands the most appropriate position for this client at this time is which of the following? A. Head of the bed flat with the patient in the supine position B. Head of the bed elevated 30 - 45 degrees in the right lateral position C. In the supine position for 30 minutes and then elevated bead of the bed at 45 degrees D. Head of the bed in semi Fowler's in the right lateral position for 60 minutes

B

What should the nurse include in a teaching plan aimed at reducing the side effects associated with taking diltiazem (Cardizem)? A. lie down after meals. B. change position slowly. C. avoid dairy products in the diet. D. take the drug with an antacid.

B

Which of the following patients would be a poor candidate for a rosiglitazone (Avandia)? A. The type 2 diabetic B. The cardiac patient in heart failure C. Any patient on insulin D. The patient with elevated high-density lipoproteins

B

Lifestyle modifications which are appropriate for patients on antihypertensive medications include: Diet and increase exercise to achieve a BMI greater than 25 Drink 4 ounces of red wine at least once per week Adopt the DASH diet Increase potassium intake

C

Receptor

Any cellular marco-molecule that a drug binds to to initiate its effects

Hydralazine

Apresoline Mechanism of action: Direct arteriole relaxation, causes reflex sympathetic effects Hypotensive effects diminish over time Monotherapy is usually not possible Reserved as 3rd or 4th line therapy in resistant HTN Should be administered with food Kinetics: Oral, IV Metabolism: genetic variations in metabolism (fast/slow metabolizers

COX Inhibitors

Aspirin (Anacin, Ecotrin) Mechanism of action: Inhibits cyclooxygenase - prevents thromboxane A2 production (platelet activator) Administer as soon as possible The effects are irreversible Therapeutic use: Nursing care: Will need to be stopped 7 days prior to surgery

As a nurse you will be responsible for completing a preoperative checklist in which you will ask your patient is they "have taken any drugs?" Even though Aspirin is available over the counter, why is it so dangerous before surgery?

Aspirin irreversibly blocks the effects of platelets so after surgery the patient would be at risk for bleeding. Because the effects are irreversible - the patient's platelets will not be able to function properly for the lifecycle of the platelet approximately 7 days.

A patient calls from home due to concern about the appearance of his insulin. He reports that when drawing up his lispro insulin he noted that the solution appeared cloudy. The nurses best response should be: A. agitate the solution and the granules should disperse. B. discard the vial as the solution should be clear. C. proceed with the injection. The process of drawing the solution into the syringe will mix the particles sufficiently. D. lispro is always cloudy. He should proceed with the injection.

B

Statin drugs

Atorvastatin (Lipitor), Simvastatin (Zocor) Action: Inhibit the enzyme necessary for cholesterol synthesis Stabilize plaques Reduce platelet aggregation /thrombi Enhance ability of the vessel to dilate

Immunomodulators

Azathioprine (Imuran) Therapeutic use: Maintain remission need to use for months prior to benefit Adverse effects: Pancreatitis Neutropenia - bone marrow suppression

The nurse is counseling a diabetic patient who is starting therapy with an acarbose (Precose). He should be educated about the potential for: A. hypoglycemia. B. flatulence. C. elevated iron levels in his blood. D. fluid retention.

B

The nurse is providing education to a group of dermatology nurses. The nurse asks the group, "Which antibiotic is frequently ordered to treat moderate to severe acne?" Select the nurses' correct response. A. Amoxicillin B. Doxycycline C. Azithromycin (Zithromax) D. Tretinoin (Retin-A)

B Currently, doxycycline and minocycline are considered the antibiotics of choice for treatment of moderate to severe acne. Amoxicillin and azithromycin are not indicated for treatment of moderate to severe acne. Although tretinoin is used to treat acne, it is a retinoid, not an antibiotic.

A patient comes to the ophthalmology clinic for a follow-up appointment. The nurse suspects that the patient is not compliant with pilocarpine for treatment of open-angle glaucoma. Which finding would support this assumption? A. The patient has miosis.(constricted pupils) B. The patient has mydriasis.(dilated pupils) C. The patient reports that her eyes are sensitive to light. D. The patient reports that her skin is sensitive to the sun.

B Dilated pupils (mydriasis) indicate that the patient is not using the eye drops, because pilocarpine causes constriction of the pupils. Miosis, or constriction of the pupils, would be expected with pilocarpine usage. Sensitivity of the eyes to light is associated with beta blockers, not pilocarpine. Sensitivity of the skin to sunlight is not associated with medications used to treat glaucoma.

The nurse is reviewing laboratory results for a patient and notes that the patient has positive results for nasal colonization by methicillin-resistant Staphylococcus aureus (MRSA). The nurse anticipates an order for which medication? A. bacitracin B. mupirocin (Bactroban) C. clindamycin (Cleocin T) D. clotrimazole (Lotrimin)

B Mupirocin (Bactroban) is used on the skin for treatment of staphylococcal and streptococcal impetigo. It is used topically and intranasally to treat nasal colonization by MRSA. The other options are incorrect. Bacitracin and clindamycin are not indicated for the treatment of MRSA; clotrimazole is an antifungal drug

The client has diabetes type 1 and receives insulin for glycemic control. The client tells the nurse that she likes to have a glass of wine with dinner. What will the best plan of the nurse for client education include? A. The alcohol could predispose you to hyperglycemia. B. The alcohol could predispose you to hypoglycemia. C. The alcohol could cause pancreatic disease. D. The alcohol could cause serious liver disease.

B Rationale: Alcohol can potentiate hypoglycemic effects in the client. Alcohol can potentiate hypoglycemic, not hyperglycemic, effects in the client. Alcohol can cause pancreatic disease, but the client's pancreas is not producing any insulin currently. Alcohol can cause liver disease, but the more immediate concern is hypoglycemia

In caring for a client with type 2 diabetes mellitus (DM), the nurse will teach the client to report: A. Normal blood glucose B. Increased blood pressure and pulse C. Increased appetite D. Absence of fever

B Rationale: Normal blood glucose is the expected outcome. An increased blood pressure and pulse are early signs of hypoglycemia. Increased appetite is an expected outcome. Clients should report a decrease In appetite. Absence of fever is an expected outcome. The nurse would have the client report an increase in temperature

The nurse is reviewing laboratory test results for a patient who is about to begin taking piperacillin. An abnormal elevation in which laboratory values would cause the nurse to notify the prescriber immediately? A. PT and PTT B. BUN and creatinine C. Hematocrit and hemoglobin D. Serum glutamic-oxaloacetic transaminase (SGOT) and alanine transaminase (ALT)

B Treatment with piperacillin requires normal kidney function. The BUN and creatinine level are the best indications of kidney function. A PT and PTT are not indicated in this situation but are indicted for heparin and warfarin. Hematocrit and hemoglobin may be needed once the patient begins treatment. SGOT and ALT are indicated for drugs that are hepatotoxic, which piperacillin is not.

A teenage boy is taking tretinoin for his acne. Part of the information the nurse should include in the teaching plan is that A. he should avoid foods heavy in salt and oils. B. the drug may cause increased redness of the skin. C. he should use abrasive cleansers to remove old skin layers. D. extremes of weather and sunlight should not bother him during therapy.

B Tretinoin may cause increased redness and drying, and the patient should avoid weather extremes, ultraviolet light, and abrasive cleansers.

A nurse is providing patient education about ciprofloxacin (Cipro). Which of the following statements by the patient would indicate a need for further teaching? "I will take my medication A. with orange juice, because acidity promotes absorption." B. with a glass of milk so it won't upset my stomach." C. at the same time as I take my vitamins." D. at least 2 hours before my antacids."

B Two nutritional drug - food interactions are significant with this class of antibiotics. This drug will chelate or bind to metal ions like calcium, iron, and multivitamins. Also milk significantly decreases absorption of this medication and therefore indicates a need for further need for teaching. There are no contraindications to taking vitamins and orange juice (except iron); no further teaching is needed. Taking antacids 2 hours prior to the medication should not affect absorption; no further teaching is needed.

A nurse is administering a metoprolol (Lopressor) to a client with CAD. Which assessment would cause the nurse to question administering the medication? A. BP 110/70 B. Apical pulse 56 C. Complaint of headache D. Potassium level 4.5 mEq/L

B is the answer because beta blockers have the potential to lowering the heart rate. A is incorrect because this is a normal blood pressure. C is incorrect because it is irrelevant. D is irrelevant to administration of a beta blocker. It can also be ruled out because the value is normal.

The nurse suspects that the client has not been taking his prescribed antihypertensive medication because the client's blood pressure remains elevated. What is the best therapeutic question the nurse can ask that will assess noncompliance? A. "I really doubt that you are taking your medication. What would you think about talking to the doctor?" B. "Taking medication is difficult for many people. What are some of your concerns about the medication?" C. "Your blood pressure is really high; do you realize the serious conse¬quences of not taking your medication?" D. "You are one of my favorite clients and I want you to be safe. Are you really taking your medication?"

B. "Taking medication is difficult for many people. What are some of your concerns about the medication?" Rationale: The most therapeutic question informs the client that compliance is difficult for many people, and does not directly challenge the client about not taking the medication. Telling the client that the nurse doubts he is taking the medication directly challenges him, and recommending that he see the physician is threatening. Telling the client his blood pressure is high and there are serious consequences is using the "scare tactic," and is non-therapeutic; the client most likely is aware of the consequences. Telling the client that he is a favorite is manipulating.

The nurse teaches a lactating woman that drug effects on the infant may be minimized by taking medications A. right after breastfeeding. B. 2 hours after breastfeeding. C. 1 hour before breastfeeding. D. right before breastfeeding.

B. 2 hours after breastfeeding.

A client's alcohol consumption would be of most concern if she was discharged home with which of the following medications? A. Acetaminophen B. A sedative C. Vitamins D. A diuretic

B. A sedative Rationale: Any alcohol will have a additive effect when combined with a sedative. Acetaminophen has the potential of causing liver injury if it is taken in high doses or with other hepatotoxic agents. Vitamins and diuretics

Before administering a scheduled narcotic, the nurse should take which of the following actions? A. Automatically cancel the order 24 hours after its administration B. Document the removal of the controlled substance from the storage area C. Count the doses of Demerol on hand if the client does not need the dose urgently D. Consult a resource book for information about controlled substances

B. Document the removal of the controlled substance from the storage area rationale: scheduled medications ianle

4. Drug X has a high therapeutic index and Drug Y has a low therapeutic index. Which drug should be monitored according to measurements of the actual blood levels achieved? A. Drug X B. Drug Y C. Both drugs X and Y D. Neither Drug X nor Y

B. Drug Y

The nurse is assessing a newly admitted client's current medications. What does the best objective data include? A. The client lists the medications that have been prescribed. B. The nurse checks the prescription bottles the client has brought to the hospital. C. The client's wife tells the nurse what medications the client has been receiving. D. The nurse asks the physician what medications the client was currently taking.

B. The nurse checks the prescription bottles the client has brought to the hospital Rationale: Objective data includes information gathered through assessment, and not necessarily what the client says or perceives. The most reliable and objective assessment by the nurse is to check the client's prescription medication bottles. A list of medications provided by the client and the client's wife is subjective, not objective data. Asking the physician what medication the client was receiving is subjective data, and the physician may not remember all the medication the client was receiving.

6. The most important factor in adverse drug reactions in the elderly population is: A. treatment with multiple drugs. B. declining renal function. C. use of over-the-counter drugs. D. use of drugs with a high therapeutic index.

B. declining renal function

A patient is taking a Category A medication during pregnancy. The nurse explains to the patient that the drug has: A. the least risk of harm to the fetus. B. no risk of harm to the fetus. C. the most risk of harm to the fetus. D. not caused birth defects in animal studies.

B. no risk of harm to the fetus.

A client asks the nurse to crush a sustained-release capsule to make it easier to swallow. Instead, the nurse withholds the dose and calls the physician for an order for a different form. The nurse explains that the major reason for refusing to crush the sustained-release capsule is that, if crushed, the coated beads of the drugs could then result in A. disintegration. B. overdose. C. malabsorption. D. deterioration.

B. overdose.

Mupirocin

Bactroban Therapeutic use: Impetigo caused by S.aureus or group A beta hemolytic streptococci Nasal carriage of S. aureus

What can be done by the nurse to prevent and/or treat complications of parenteral nutrition?

Because patients receiving parenteral nutrition are at constant risk of infection, aseptic technique should be employed during catheter insertion and during preparation and administration of solutions. Using a central venous catheter that has been impregnated with antiseptics can further reduce risk of infection. If the patient develops a fever, sepsis should be suspected. To assess for sepsis, blood cultures should be drawn from the tip of the IV line as well as from a separate venous site. The nurse should also constantly monitor for metabolic disturbances such as hyperglycemia, hypoglycemia, hyperlipidemia, and elevation of BUN as well as fluid and electrolyte status.

Anticholinergic Agents

Benztropine (Cogentin), Trihexyphenidyl (Artane) Therapeutic use: Reduce tremor and rigidity No effect on bradykinesia Mechanism of action: Block muscarinic cholinergic receptors in the striatum

Class 3

Betamethasone 17-valerate

Kernicteris

Bilirubin deposits in the brain

Stimulant laxatives

Bisacodyl (Dulcolax) Adverse effects: Abdominal cramping, nausea, diarrhea Fluid and electrolyte loss

Thrombin Inhibitors

Bivalirudin (Angiomax) Mechanism of action: Direct inhibition of thrombin Therapeutic use: Used as an adjunct with ASA Prevents clot after angioplasty Pharmacokinetics: 25" half life

Non steriodal (Ibuprofen)

Block prostaglandins

Antagonist

Blocker

NSAIDs (Ibuprophen, Naproxen)

Blocks COX Mechanism of action: Non selective means it effects COX I and COX II Inhibit COX I - gastric erosion and ulceration Bleeding tendencies - irreversibly blocks COX from forming thromboxane Also inhibits COX II - suppresses inflammation, alleviates pain, reduces fever

Nolxone (Narcan)

Blocks access to opiate receptor site

38. A physician prescribes the potassium sparing diuretic spironalactone (Aldactone). In which of the following conditions should the nurse discontinue the drug? A. Gout attacks B. Elevated blood glucose C. Serum potassium level greater 5.2 mEq/mL D. Migraine headaches

C

A A patient in the ICU requires intravenous insulin. The nurse is aware that: A. any form of insulin can be used IV at the same dose as that ordered for subcutaneous administration. B. insulin should never be given IV and this order should be questioned. C. only regular insulin can be administered IV. D. Regular insulin can be mixed with long acting insulin in an IV solution. .

C

A Insulin glargine has been ordered for a diabetic patient on the nursing unit. The nurse will expect to administer this: A. approximately 15-30 minutes before each meal. B. in the morning and at 4 p.m. C. once daily. D. after meals and at bedtime.

C

A patient received NPH 20 units subcutaneously at 8:00 AM. The nurse should assess the patient for hypoglycemia reaction at: A. 10:00 AM B. 11:00 AM C. 5:00 PM D. 11:00 PM

C

Assuming the patient eats breakfast at 8:30 AM, lunch at noon, and dinner at 6:00 PM, he or she is at highest risk of hypoglycemia following an 8:00 AM dose of NPH insulin at 10:00 AM 2:00 PM 5:00 PM 8:00 PM

C

Azole

Itraconazle (Sporanox) Ketoconazole (Generic) - Topical cream Clotrimazole (Mycelex) - Vaginal suppository or cream Fluconazole (Diflucan) - Oral, vaginal Therapeutic use: Dermatophyte or candida infections Adverse effects: Oral -azoles can cause liver injury and cardiac suppression

The nurse is providing education on timolol (Timoptic) to a patient who has been diagnosed with open-angle glaucoma. Which statement by the patient best demonstrates the need for further teaching? A. "I should take my pulse prior to each application." B. "I may experience blurred vision." C. "I should cover my skin when I am out in the sun." D. "I may experience stinging and dry eyes."

C The nurse is providing education on timolol (Timoptic) to a patient who has been diagnosed with open-angle glaucoma. Which statement by the patient best demonstrates the need for further teaching? A. "I should take my pulse prior to each application." B. "I may experience blurred vision." C. "I should cover my skin when I am out in the sun." D. "I may experience stinging and dry eyes."

A newly admitted patient is going to be started on a course of antibiotic therapy with penicillin. The nurse understands that, while taking the patient's health history, a priority assessment is questioning the patient about A. a history of seizure activity. B. the cardiac history. C. any previous allergy to penicillin. D. any recent urinary tract infections (UTIs).

C All patients who will be receiving penicillin therapy should be asked if they have penicillin allergies. A history of seizures and a cardiac history are not contraindications to receiving penicillin. A UTI also is not a contraindication to antibiotic therapy with penicillin.

A father calls because his son has head lice. He reports that he has used "that special medicated shampoo three times, but nothing is happening. Now what?" What will the nurse advise first? A. Get a prescription for a second product, malathion. B. Try one of the lotion products that remain on the scalp for 8 hours. C. Be sure that he is using a nit comb to remove nits from the hair shafts. D. Comb through the hair with mineral oil to loosen the lice from the hair shafts.

C Before trying another product, ensure that he is performing the regimen correctly. Because he only mentioned shampooing, ensure that after each shampoo he is using a nit comb to remove nits, or eggs, from the hair shafts.

A patient is receiving intravenous (IV) vancomycin (Vancocin). A unique complication of rapid IV infusion of vancomycin is A. pleural effusion. B. angioedema. C. "red man" syndrome. D. neurotoxicity.

C Rapid infusion of vancomycin can cause flushing, rash, pruritus, urticaria, tachycardia, and hypotension, known collectively as "red man" syndrome. Pleural infusion is unlikely with IV vancomycin. Angioedema is not associated with vancomycin administration. Neurotoxicity is not associated with rapid infusion of IV vancomycin

A client with diabetes mellitus is taking oral agents, and is scheduled for a diagnostic test that requires him to be NPO (nothing by mouth). What is the best plan of the nurse with regard to giving the client his oral medications? A. Administer the oral agents with a sip of water before the test. B. Administer the oral agents immediately after the test. C. Notify the physician and request orders. D. Notify the diagnostic department and request orders.

C Rationale: It is best to notify the client's physician and request orders. The client should not receive the medication during NPO (nothing by mouth) status unless directed by the physician. The medications should not be given upon return unless the physician orders this; the client may still need to be NPO (nothing by mouth). The radiologist in the diagnostic department might give orders, but it would be best to check with the client's physician first.

The client injects his insulin as prescribed, but then gets busy and forgets to eat. What will the best assessment of the nurse reveal? A. The client will be very thirsty. B. The client will complain of nausea. C. The client will have moist skin. D. The client will need to urinate.

C Rationale: Moist skin is a sign of hypoglycemia, which the client would experience if he injected insulin and did not eat. Thirst is a sign of hyperglycemia; the client would experience hypoglycemia if he did not eat. Nausea is a sign of hyperglycemia; the client would experience hypoglycemia if he did not eat. Increased urination is a sign of hyperglycemia; the client would experience hypoglycemia if he did not eat.

A patient has just received a prescription for a 10-day course of penicillin for the treatment of strep throat. What patient education is essential for the nurse to provide? A. Family members who have the same symptoms should also receive treatment. B. The agent should be discontinued as soon as symptoms resolve to prevent allergic reactions. C. The entire course of the drug must be taken to prevent the recurrence of infection and potential resistance. D. The medication should be discontinued as soon as symptoms resolve to reduce the risk of resistance.

C The nurse should instruct the patient to take the entire prescribed course of medication to prevent the recurrence of infection and potential resistance. Treatment of other family members with the same regimen is not always indicated, and their symptoms may be related to something else. The patient should always be advised to finish the entire course of antibiotics for maximum effectiveness; discontinuing the agent early does not prevent allergic reactions. Discontinuing the agent as soon as symptoms resolve does not reduce the risk of resistance.

A nurse is preparing a teaching plan for a client that is prescribed acarbose. What instruction should the nurse include in the teaching plan for this patient? A. avoid drug administration in case of a skipped meal B. report respiratory distress or muscular aches to the primary care provider. C. keep a source of glucose ready for signs of low blood glucose. D. take the drug at different times during the day

C is correct. A is incorrect because this advice is appropriate for a patient taking repaglinide (Prandin) or Nateglinide (Starlix) which bear a risk of hypoglycemia. B is incorrect - this instruction is important in patient receiving metformin. D is incorrect because these medications should be taken at the same time each day.

In which of the following scenarios would the nurse consider antibiotic therapy to be prophylactic? (Select all that apply.) A. A patient with neutropenia and cancer who experiences fever and cough B. A patient with a fever of unknown origin C. A patient undergoing an operation for colon resection D. A patient with an artificial heart valve who is scheduled for hernia repair E. A patient with a new diagnosis of endocarditis

C, D Prophylactic antibiotic therapy is appropriate for patients undergoing surgery on the GI tract, such as a colon resection, and certain other surgeries, such as cardiac surgery, peripheral vascular surgery, and orthopedic surgery. It also is appropriate for a patient with an artificial heart valve who is scheduled for a hernia repair, because it reduces the patient's risk of bacterial endocarditis. Not enough information is provided to determine whether the patient with neutropenia, cancer, fever, and cough needs antibiotic therapy. A patient with a fever of unknown origin may or may not need a broad-spectrum antibiotic, depending on the diagnosis. A patient with a new diagnosis of endocarditis does not need prophylactic antibiotic therapy, because the patient already has an infection. Antibiotic therapy would be considered treatment. Prophylaxis is given to prevent an infection.

Lifestyle modifications which are appropriate for patients on antihypertensive medications include: A. Diet and increase exercise to achieve a BMI greater than 25 B. Drink 4 ounces of red wine at least once per week C. Adopt the DASH diet D. Increase potassium intake

C. Adopt the DASH diet

After hearing the beginning of the shift report, the nurse wants to perform medication reconciliation for the clients assigned for the day. To do this properly, the nurse would do which of the following? A. Ensure that all medications were charted from the previous shift B. Ensure that all medications have been signed off in the chart since the last 24-hour chart check C. Compare the current medication list against the client's medication list from home and any admission and transfer orders D. Ask each client if he or she plans to refuse any medications ordered for the shift

C. Compare the current medication list against the client's medication list from home and any admission and transfer orders

The nurse working in a newborn nursery understands that which factor places the neonate at risk regarding pharmacokinetics? A. Hyperperistaltic bowel B. Dysfunctional musculoskeletal system C. Immature renal functioning D. Efficient temperature regulation

C. Immature renal functioning

The nurse would document which of the following manifestations as characteristic of a mild allergic reaction to a drug? A. Diarrhea B. Dysuria C. Pruritus D. Lymphadenopathy

C. Pruritus

Calcium channel blockers

Verapamil (Calan, Isoptin) Diltiazem ER (Cardizem CD, Cartia XT) Method of action: Effect movement of calcium into cells Work on vascular smooth muscle Relax peripheral arterioles Therapeutic Use: Variant angina Stable angina

Pancreatic Enzymes

Pancrelipase (Pancreze, Creon) Mechanism of action: contains lipase, amylase, and protease. acts locally in LGI tract Adverse effects: fairly uncommon. N,V,D Contraindications: pork allergies Nursing care: Assess for pork allergies Assess for high levels of uric acid

2. A patient has a medical history of liver disease from alcoholism. The nurse knows that this will have which of the following effects? A. The patient will have drug tolerance B. The patient will experience drug interactions C. The patient is susceptible to an additive effect D. The patient will have a paradoxical effect

C. The patient is susceptible to an additive effect

3. A patient is prescribed cimetidine (a histamine H2 antagonist) to treat a gastric ulcer. The nurse will teach the patient that this drug will: A. react with gastric acid to form a thick paste and adhere to the ulcer area. B. activate mucous secretion by the parietal cells to protect the gastric lining. C. inhibit the action of histamine at receptor sites and block gastric acid secretion. D. neutralize gastric acid by inactivation of pepsin

C. inhibit the action of histamine at receptor sites and block gastric acid secretion.

A client with CAD is prescribed aspirin. The client states "I had an ulcer last year." Which action by the nurse would be most appropriate? A. ask the client if they told their medical provider. B. instruct the client to take enteric coated aspirin. C. determine if the client is taking any other medications. D instruct the client about the risks of bleeding with ulcers.

C. is correct because it is important to see if they are taking any other medication and they also may be treated with misoprostol (Cytotec). Misoprostol prevents ulcers in patients taking ASA. A is incorrect because if you were concerned you would notify the provider yourself. B is incorrect because a nurse would not be changing an order on their own - this is out of the scope of nursing. D is incorrect because although it good information it doesn't specifically solve the problem.

When administering famotidine (Pepcid) it is important to assess for which side effects?

CNS effects are common in the elderly, In general these are avoided due to this problem. Central nervous system effects such as confusion or hallucinations.

Hormone therapy

Calcitonin (Miacalcin) Mechanism of action: Chemically made version of the hormone calcitonin Adverse effects: Rare Rhinitis (with nasal spray); not with injectable Nursing care:

Partial agonist

Partially stimulates (Chantix)

ACE Inhibitors

Captopril (Capoten) enalapril, lisinopril Mechanism of action: Inhibits angiotensin I conversion to II (active) Helps with formation of vasodilating substances bradykinins and prostaglandins Role in kidney protection: Preferentially dilate efferent arterioles-relieves intraglomerular pressure Indications: HTN, HF

Sucralfate

Carafate Mechanism of action: "Sticky" product that adheres to ulcer base. Effects lasts for about 6 hours Pharmacokinetics: administered orally absorption is minimal - 90% feces Adverse effects: constipation Nursing care: Impairs absorption of other medications take other medications 2 hours before

Cephalosporins

Change cell wall causing contents to spill out

Bile acid binding resins

Cholestyramine (Questran) Colesevelam (Welcol) Interactions: Delay or prevent absorption of medications Adverse effects: Constipation Abdominal - pain, flatulence, nausea Steatorrhea Deficiencies of vitamins A, D, E, K, folic acid Nursing: Take 1 hour before OR 4 hrs after other meds Don't let it sit Mix in at least 6 ounces Diet recommendations

H2 Receptor Antagonists

Cimetadine (Tagamet) Famotidine (Pepcid) Mechanism of action: H2 receptors are located on parietal cells Promote secretion of gastric acid Decreases volume of gastric juices Reduces hydrogen ion concentration Pharmacokinetics: Cross blood brain barrier CNS side effects can happen

Fluoroquinolones

Ciprofloxacin Cipro Ofloxacin Floxin Therapeutic use: Respiratory, urinary tract, joint / soft tissue infection Mechanism of action: Inhibits bacterial DNA gyrase - inhibits replication Adverse effects: Sun sensitivity GI: Nausea, vomiting, diarrhea, abdominal pain Candida infections of the throat CNS: dizziness, HA, restlessness and confusion Tendon rupture: medication interrupts extracellular matrix of cartilage. Not given to children

Class 4 (strongest)

Clobetasol 17-propionate

Alpha2 Agonists

Clonidine (Catapress) Methyldopa Mechanism of action: Central stimulation of alpha 2 receptors decreases sympathetic response from the brainstem e Peripheral vasodilation occurs by relaxing peripheral arterioles and veins Indications: BP, alcohol or opiate withdrawal

Anitussives

Cough medicine Dextromethorphan

Warfarin

Coumadin Indications: DVT/PE Atrial Fibrillation Myocardial Infarction Mechanical Heart Valves Action: Antagonists Vitamin K Kinetics: Steady state Warfarin therapy is started - coadministered with Heparin or LMWH

Mast Cell Stabilizer

Cromolyn (Intal), nedocromil (Tilade) Mechanism of action: Suppresses inflammation Stabilizes cytoplasmic membrane of mast cell Therapeutic Uses: Chronic asthma Exercise induced bronchospasm

Misoprostol

Cytotec Therapeutic use: Only use - gastric ulcers from NSAIDs Mechanism of action: Promote secretion of bicarbonate and protective mucus Prevents NSAID ulcers by replacing endogenous prostaglandins

Cyclophosphamide Alkylating Agents

Cytoxin Mechanism of action: cell cycle nonspecific Forms cross links between DNA strands Nursing care: Hydration Report palpitations Teach reliable birth control, no breast feeding

A A Type 1 diabetic on insulin reports that he is taking propranolol for hypertension, this provokes concern that: A. The beta blocker can produce insulin resistance. B. the two agents used together will increase the risk of ketoacidosis. C. propranolol will increase insulin requirements due to receptor blocking. D. the beta blocker can mask the symptoms of hypoglycemia.

D

Before administering an intermittent tube feeding through a nasogastric tube, the nurse assesses gastric residual. The nurse understands that this procedure is important to: A. Confirm proper placement of the nasogastric tube placement B. Observe the digestion of formula C. Assess fluid and electrolyte status D. Evaluate absorption of the last feeding

D

Rationale: D; Infiltration anesthesia is commonly used for minor surgical procedures. It involves injecting the local anesthetic solution intradermally, subcutaneously, or submucosally across the path of nerves supplying the area to be anesthetized. The local anesthetic may be administered in a circular pattern around the operative field. The others types are not appropriate for this injury.

D

The nurse is administering timolol (Timoptic) eye drops to a patient who has been taking these drops for years. What potential systemic effects, if any, will the nurse monitor the patient for after this medication is given? A. Hypotension B. Bradycardia C. Dry mouth D. Increased heart rate

D Although systemic effects associated with the use of sympathomimetic mydriatics such as dipivefrin are uncommon, they are theoretically possible, especially with the use of larger doses or prolonged drug therapy. They include cardiovascular effects, such as extrasystoles, tachycardia, and hypertension. Other effects that may be noticed are headache and faintness.

A patient who recently was discharged from the hospital and prescribed ciprofloxacin (Cipro) phones the prescriber's office complaining of pain above the heel while walking or stretching. The nurse expects the patient to be instructed to A. try isometric exercises prior to walking or stretching. B. avoid the treadmill and apply ice to area. C. continue the medication and watch for signs of redness or swelling. D. discontinue the medication and avoid weight bearing on the ankle.

D Ciprofloxacin is a fluoroquinolone, which puts this patient at risk for tendon rupture. It is essential that the nurse tell the patient to report any tendon tenderness, which may precipitate tendon rupture. The nurse also should instruct the patient to discontinue the medication and avoid weight bearing on the ankle. The medication must be discontinued, which is not stipulated in the other options.

The physician writes orders for the client with diabetes mellitus. Which order would the nurse validate with the physician? A. Use Humalog insulin for sliding scale coverage. B. Administer regular insulin 30 minutes prior to meals. C. Metformin (Glucophage) 1000 mg per day in divided doses. D. Lantus insulin 20U BID.

D Rationale: Lantus insulin is usually prescribed in once-a-day dosing so an order for BID dosing should be validated with the physician. Humalog insulin can be prescribed for sliding scale coverage. Regular insulin is administered 30 minutes before meals. Metformin (Glucophage) is often prescribed in divided doses of 1000 mg per day.

The nurse has finished teaching a client with diabetes mellitus how to administer insulin. The nurse evaluates that learning has occurred when the client makes which statement? A. "I should use the abdominal area only for insulin injections." B. "I should provide direct pressure over the site following the injection." C. "I should check my blood sugar immediately prior to the administration." D. "I should only use a calibrated insulin syringe for the injections."

D Rationale: To ensure the correct insulin dose, a calibrated insulin syringe must be used. Insulin injections should also be rotated to the arm and thigh, not just the abdominal area. There is no need to apply direct pressure over the site following an insulin injection. There is no need to check blood glucose immediately prior to the injection.

The nurse is caring for a patient receiving gentamicin (Garamycin) intravenously (IV). What statement by the patient would most concern the nurse? "I am experiencing A. dizziness." B. ear pain." C. tone deafness." D. high-pitched ringing in my ears."

D The first sign of impending cochlear damage is high-pitched tinnitus (ringing in the ears). As injury to cochlear hair cells proceeds, hearing in the high-frequency range begins to decline. The first sign of vestibular damage is headache, which may last 1 to 2 days. Nausea, unsteadiness, dizziness, and vertigo then begin to appear.

A patient's spouse phones the clinic and reports, "My husband used his dorzolamide (Trusopt) eye drops, and his eyes and eyelids are very red. What should we do?" Select the nurse's best response. A. "This is a normal and transient reaction. Continue the drops as prescribed." B. "This is a common side effect and means that the drug is working." C. "You are using the drug too frequently. Skip every other dose for the next week." D. "Stop using the drops and bring him to the office as soon as possible."

D The most common side effects of dorzolamide are ocular stinging and a bitter taste. Some patients experience conjunctivitis and lid reactions, which are allergic reactions. If these occur, the patient should stop using the drops and contact the prescriber. The symptoms are related to an allergic reaction; they are not side effects that may subside. Skipping doses will not resolve the situation

A patient is started on latanoprost (Xalatan) for treatment of glaucoma. The nurse provides patient education on the medication. What statement by the patient best demonstrates understanding of latanoprost? A. "The drug may produce drowsiness, so I will use caution when driving or operating dangerous machinery." B. "Nausea and vomiting are common side effects, and antiemetic drugs may be needed." C. "Hypotension is common with this medication, so I should change positions slowly." D. "The drug may cause a harmless, heightened brown pigmentation of the iris."

D The most significant side effect of latanoprost is a harmless, heightened brown pigmentation of the iris; this statement indicates understanding. Latanoprost is not known to cause drowsiness; further teaching is needed. Nausea and vomiting are not common side effects; further teaching is needed. Latanoprost does not cause hypotension; further teaching is needed.

The nurse is providing education to a 24-year-old female patient with severe nodulocystic acne vulgaris. The nurse anticipates treatment with isotretinoin (Accutane). The nurse is explaining the iPLEDGE program to the patient. Which statement by the patient demonstrates a need for further teaching? A. "Pregnancy must be absolutely ruled out prior to starting the medication." B. "I will receive oral and written warnings about the high risk of fetal harm if the medication is taken during pregnancy." C. "I must use two effective forms of birth control." D. "I will implement birth control measures at least 3 weeks prior to starting the medication."

D The patient must begin using birth control measures at least 1 month prior to initiation of therapy and for 1 month after stopping therapy; this statement indicates that further teaching is needed. The other responses are consistent with the iPLEDGE program; further teaching is not needed

A client diagnosed with CAD is taking Aspirin 81 mg per day. What is the rationale for taking this medication? A. Daily aspirin decreases angina. B. Baby aspirin prevents gastric distress from CAD. C. Aspirin thins the patient's blood. D Aspirin prevents platelet aggregation.

D is correct. A is false, because angina is the reason a patient takes aspirin. B is incorrect because aspirin can cause gastric distress. D in incorrect because aspirin does not thin the blood.

Instructions to a client regarding self-administration of oral enteric-coated tablets should include which of the following statements? A. "Avoid any other oral medicines while taking this drug." B. "If swallowing this tablet is difficult, dissolve it in 3 ounces of orange juice." C. "The tablet may be crushed if you have any difficulty taking it." D. "To achieve best effect, take the tablet with at least 8 ounces of fluid."

D. "To achieve best effect, take the tablet with at least 8 ounces of fluid."

The physician prescribes an oral medication for the client. Which assessment is vital prior to administering this medication? a. The client's understanding of the medication b. The client's eyesight c. The client's breath sounds d. The client's ability to swallow

D. (The client's ability to swallow Rationale: The ability of the client to swallow is a safety issue to prevent aspiration of the medication. The client's understanding is important, but not a priority. The client's eyesight is not significant. The client's allergies are important, but if the client cannot swallow the medication, then the allergies are not significant.)

Which of the following describes the process of the movement of drug molecules from the site of entry into the body? A. Biotransformation B. Distribution C. Excretion D. Absorption

D. Absorption

A nurse is evaluating the kidney function of an 82-year-old patient before the administration of medications. Which laboratory result would provide the best index of renal function? A. Serum creatinine B. Blood urea nitrogen (BUN) C. Urinalysis D. Creatinine clearance

D. Creatinine clearance

7. Digoxin has a half-life of 36 to 48 hours. Because of the length of the half-life, the nurse expects dosing to be: A. Four times a day B. Three times a day C. Two times a day D. Once a day

D. Once a day

LIndane

Kweli Adverse effects: Recent concern over Neurotoxicity of lindane Hematotoxicity Patient management: Pediculosis - an application is applied to the affected area for 4 minutes then rinsed off; retreat in one week if living lice remain Scabies - a single application from the neck down; left on for 8 - 12 hours Avoid use in infants, children and pregnant women

A patient has been taking a narrow therapeutic index medicine for one week. What is the most significant implication for the nurse who is caring for this patient? A. The drug will be ineffective because it will have difficulty reaching therapeutic level. B. The patient's body will maintain therapeutic levels easily. C. The patient's kidney function may be compromised. D. The patient should be monitored carefully for toxicity.

D. The patient should be monitored carefully for toxicity.

For the treatment of Graves' disease, ___ is used as adjunctive therapy to suppress tachycardia.

For the treatment of Graves' disease, propanolol ___ is used as adjunctive therapy to suppress tachycardia.

Two hours after taking a dose of penicillin, a patient arrives in the emergency department complaining of tightness in the throat, pruritus, and red wheals. During the physical assessment, the patient develops difficulty breathing, respirations 36/min, blood pressure 90/42, pulse rate of 120/min. The priority nursing action would be to A. administer diphenhydramine (Benadryl), because the patient is experiencing a moderate allergic reaction that should improve shortly B. sit the patient up in bed, administer oxygen until the symptoms subside, and notify the prescriber. C. question the patient about any previous allergy to penicillin and report the symptoms to the prescriber. D. call for assistance, apply oxygen, administer epinephrine as ordered, and notify the prescriber.

D. call for assistance, apply oxygen, administer epinephrine as ordered, and notify the prescriber.

9. A nurse receives a handwritten medication order in which the writing is not legible. The nurse should: A. consult with the charge nurse to verify the order. B. discuss the order with the pharmacist. C. check with the patient to determine the correct medication. D. contact the prescriber to clarify the order.

D. contact the prescriber to clarify the order.

Neutropenia

Decreased number of granulocytes (type WBC) Measures the first line of defense against infection Absolute neutrophil count (ANC) = number of granulocytes in the blood ANC = total WBC x (% polys + % bands) example: 5000 x ( 40% + 10%) = 2,500 ANC < 500 = Severe risk of infection

Thrombocytopenia

Decreased number of platelets Risk of bleeding increases below 50,000 Nursing care: Assess head to toe for bleeding/petechiae Monitor platelet count Teach patient to report signs of bleeding or increased petechiae Transfuse as ordered

Anemia

Decreased number of red blood cells Transfusions given when Hgb < 8, Hct < 24 or patient is symptomatic Tolerance for fatigue varies greatly Most common side effect from chemotherapy

Penicillin

Destroy cell wall of virus. Releases betalactomase, betalactomase will kill antibiotic monitor creatinine, creatinine clearance

Glucocorticoids

Dexamethasone (Decadron) Predinsone Therapeutic use: Induce remission in UC and Crohn's disease Not for long term maintenance Adverse effects: Adrenal suppression Osteoporosis Risk for infection Cushingoid syndrome

Phenytoin

Fosphenytoin (Cerebxy), (Dilantin) Mechanism of action: selective inhibition of sodium channels Therapeutic uses: Partial and tonic clonic seizures Drug Interactions: Urine may turn a harmless pink color

A 45 year old female with a 10 pack year history of smoking is started on antihyperlipedemic therapy. The client states "this is great! now I can eat whatever I want." Describe some teaching points that should be made with this client.

Dietary measures are always important in treating patients with hyperlipedemia. Foremost, smoking cessation is important. Other non pharmacologic measures that should be emphasized include: weight loss, exercise, and decreased saturated fats. Female gender doesn't always protect people from heart disease.

Anaphylaxis

Difficulty breathing, wheezing, flushed or pale

Antidiarrheal Agents

Diphenoxylate (Lomotil) Activate opioid receptors in the GI tract, decrease intestinal motility, slow intestinal transmit Contraindications: severe liver disease, narrow angle glaucoma, diarrhea from pseudomembranous colitis

Surfactant laxatives

Docusate sodium (Colace

Acetylcholinesterase Inhibitors

Donepezil (Aricept), galantamine (Reminyl), rivastigmine (Exelon) Mechanism of action: Inhibits acetylcholinesterase Side effects: anorexia, nausea and vomiting, constipation Adverse effects: Seizure, bradycardia, orthostatic hypotension, cataracts, MI, heart failure

Heparin

Enoxaparin (Lovenox), Dalteparin (Fragmin) Mechanism of action: Inactivates thrombin and other clotting factors (Intrinsic cascade) Therapeutic use: Adjunctive with acute MI Prophylaxis of deep vein thrombosis Treatment of DVT or pulmonary emboli Acute CVA Goal of therapy: Achieve a PTT 60 - 90 seconds Antidote: protamine sulfate

Epoietin alfa

Epogen Procit Mechanism of action: Stimulates precursors to red blood cells Indications: Chemotherapy induced anemia Renal failure induced anemia Adverse effects: Hypertension Blood clot N, V, constipation

Ergot Alkaloids

Ergotamine (Ergostat) Dihydroergotamine (Migrainal) Mechanism of action: Agonist activity on serotonin receptors Blocks inflammation of trigeminal vascular system

Bisphosphonates

Etidronate (Didronel) Alendronate (Fosamax) Mechanism of action: Adverse effects: Upper GI irritation / acid reflux (common) Esophageal ulceration Hypocalcemia Jaw necrosis (IV - higher uptake) Assessment: patients must be able to sit up for at least 30 minutes when taking this medication Monitor for calcium level 9 - 10 mg / dL Report signs of hypocalcemia Trousseau's sign: pump a BP cuff up; a positive sign occurs when the hand goes into spasm Chvostek's sign: tap on facial nerve; a positive positive sign occurs when the lip twitches in the corner

Flutamide Antiandrogen

Eulexin Mechanism of action: Blocks androgen uptake Inhibiting binding of androgen in target tissues Indications: prostate cancer Nursing care: Assess for liver injury Monitor AST, ALT Stimulates cross gender sexual characteristics Impotence

Incretin Mimetics

Exenatide (Byetta) Mechanism of action: Binds to glucagon like peptide (hormone) Slows gastric emptying Stimulates release of insulin Inhibits post prandial release of glucagon Contraindications: Not to be used in pregnancy Nursing care: Injection in abdomen; within 60 minutes before morning and evening meals NOT insulin

True or false: Anticoagulant drugs are used to break down or lyse clots present in the venous system.

False; anticoagulant drugs will prevent further clots from forming and will not dissolve existing clots

True or false: Antiplatelet drugs are used to prevent venous thrombosis.

False; antiplatelet drugs are used to prevent arterial thrombosis.

Epidural Catheters

Fewer and less severe side-effects of the local anesthetic, which can include: Weakness (motor nerve block) Numbness (sensory nerve block) Low blood pressure (autonomic nerve block) Local anesthetic toxicity (ringing in ears, metallic taste, lip and oral numbness, light headedness, and very rarely seizures, cardiac disturbances, coma and even death)

Metronidazole

Flagyl Therapeutic uses: acute intestinal amebiasis (anaerobic bacteria) and protozoal infections (Trichomonias) Adverse effects: Peripheral neuropathy Neurological (optic neuropathy, seizures) Dysuria Nausea, epigastric distress

Antifungal

Fluconazole Mechanism of action: Inhibits synthesis of ergosterol needed for cytoplasmic membrane Adverse effects: Cardiac suppression - negative inotropic effect Liver injury - liver failure MOnitor AST, ALT for elevation Not used for Hepatitis

Steriods

Fluticasone (Inhaled) Flovent Prednisone. Methyprednisolone Medrol Mechanism of action: suppress inflammation Decrease synthesis / release inflammatory mediators Decrease infiltration / activity of inflammatory cells Decrease edema of airway mucosa - reduce bronchial hyperactivity Decrease mucous production Therapeutic use: Prophylaxis of chronic asthma COPD - inhaled may improve airflow

Aminoglycosides

Gentamycin Garamycin Amikacin Amikiin Mechanism of action: bactericidal Kill is related to concentration Therapeutic use: Serious gram negative bacteria Pharmacokinetics: Interpatient variability: Different patients will receive same dose and achieve different levels Adverse effects: Ototoxicity- all accumulate in the inner ear Cause cell injury - impair hearing and balance Causes direct damage to sensory sells in cochlea Damage d/t trough levels - prolonged exposure Risk increased with: renal impairment, diuretics Doses for greater than 10 days Vestibular damage - headache 1-2 days Lose high frequency first Nephrotoxicity: injure cells of proximal renal tubules Correlates to total cumulative exposure Proteinuria, casts in the urine, production of dilute urine, elevations of Cr and BUN Drug Interactions: Ototoxic drugs: loop diuretic (lasix) Nephrotoxic drugs: NSAIDs, ASA, Vanco Nursing care: Monitoring serum levels trough is very important Timing is key - want trough as close to 0

Biguanides

Glucophage (Metformin), Glucophage XR Mechanism of action: Decreases production of glucose in liver Suppression of gluconeogenesis Enhances glucose uptake in muscle Adverse effects: Decreased appetite, nausea, and diarrhea Lactic acidosis: Rare medical emergency Fluid and water retention

Alpha-Glucosidase Inhibitors

Glyset and Precose Mechanism of action: Delays absorption of dietary carbohydrate Enzyme on brush border of intestine Therapeutic use: Type 2 diabetes used along and in combination with insulin

Expectorants

Guaifenasin Mucinex

The nurse is monitoring a patient on IV heparin and warfarin (Coumadin). The laboratory results are as follows: INR 1.8; PTT 80 and control 35. What should the nurse do?

Heparin is given intravenously until the warfarin reaches steady state which usually takes 3-5 days. A PTT of 80 is therapeutic. The INR of 1.8 is almost therapeutic. So the warfarin would be given and the heparin should be continued.

How does the nurse know the outcome of heparin therapy? Hint: why is the patient taking the medication?

Heparin is used to prevent DVT so make sure the patient doesn't have any signs of DVT. Signs of DVT include: warm limb, redness, and wider calf. The labs are used to make sure the patient is at a therapeutic level.

Drug schedule 1

Heroin, No medical use

Cytochrome P450

How liver breaks down medications.

Thiazide diuretics

Hydrochlorothiazide (Hydrodiuril) Chlorothiazide (Diuril) Related: metolazone (Zaroxolyn) Modest diuresis Acts on the early segment of the distal convoluted tubule

Why is it important to know what the patients creatinine level when performing a pre administrations prior to administering a diuretic?

Hydrochlorothiazide requires luminal perfusion (adequate creatinine level) to provide diuresis. In contrast, loop diuretics will provide robust diuresis even if the patient is in renal failure.

Hypoglycemia is defined as blood glucose of less than __________.

Hypoglycemia is defined as blood glucose of less than __90 ________.

Fastest and slowest absorption rate

IV fastest and oral slowest

What special precautions should be taken if a patient's TPN become interrupted?

If the TPN is interrupted for any reason - usual protocol is to infuse dextrose 10% so the patient will not become hypoglycemic. This happens because the patients body secretes insulin from the pancreas in response to a certain amount of glucose - and when the source of glucose is stopped (like the TPN was never reordered or the bag fell on the floor) the patient may become hypoglycemic. Nursing responsibilities also include monitoring for correct placement of the catheter to prevent catheter-related complications such as pneumothorax and central venous thrombosis or phlebitis.

Decongestants

Imitate sympathetic nervous system Phenylephrine (Neo synephrine) Pseudoephedrine (Sudaphed) Oxymetazoline (Afrin) Mechanism of action: Relieve stuffy nose only Therapeutic uses: Allergies, sinusitis

What is a percutaneous coronary intervention? And what drugs are useful in treating the patient?

In PCI a balloon is inserted into an artery. The balloon is then inflated which compresses the blockage. When the balloon is blown up, the narrowed coronary artery is widened. A wire stent is then inserted which keeps open the lumen. A major problem with PCI and stent placement is reocclusion. In the immediate post procedure time period drugs like abciximab (Reopro) and tirofiban (Aggrastat) are useful to prevent acute re-occlusion of the stent.

Endocarditis

Inflammation of the heart

Antiicholinergic

Ipatropium (Atrovent HFA) Ipatropium with albuterol (Duoneb) for nebulization Mechanism of action: Blocks muscarinic (cholinergic) receptors bronchus Bronchodilation occurs Adverse effects: Dry mouth Irritation of pharynx May raise IOP if level high enough

Closed Angle Glaucoma

Iris "bows" and blocks the trabecular network where the fluid normally exits

64. Why are nitrates transdermal and oral nitrates not scheduled around the clock?

It is important that the patient has a "nitrate" free period so they will not become tolerant to the medication. The nitrate free period usually occurs when the patient is least likely to have an angina attack - for example, during the nighttime.

Why are nitrates transdermal and oral nitrates not scheduled around the clock?

It is important that the patient has a "nitrate" free period so they will not become tolerant to the medication. The nitrate free period usually occurs when the patient is least likely to have an angina attack - for example, during the nighttime.

Digoxin (Lanoxin, Digitek)

Lanoxin Digitek Mechanism of action: Positive inotropic effects on heart - increases force Inhibits Na, K pumps - increasing Ca within heart Also effects pumps in renal tubules - decreases tubular absorption of sodium - which in turn suppresses renin release Also decreases activation of sympathetic nervous system Decreases conduction velocity

Proton Pump Inhibitor

Lansoprazole (Prevacid), Omeprazole (Prilosec) Pantoprazole (Protonix) Therapeutic use: Gastric and duodenal ulcer Mechanism of action: most effective! Causes irreversible inhibition of H+, K+, ATPase Pharmacokinetics: Plasma half life is short - effects are irreversible Therapeutic use: Hypersecretory conditions (Zollinger Ellison) Gastric ulcer GERD Adverse effects: Headache, diarrhea, nausea and vomiting Long term use risk of gastric cancer is due to hypersecretion of gastrin Gastrin causes hyperplasia of gastric epithelial cells Vitamin B12 malabsorption long term use Nursing care: Capsules and tablets - do not crush or open IV pantoprazole or lansoprazole use IV filter Instruct patient to consume adequate amounts of Ca, Mg, iron, folic acid and B12

A client is taking lispro and lente for diabetes. What are the onset, and duration? What are the key teaching points the nurse would include?

Lispro: Short acting • Make sure that there is food available for you to eat prior to administering it • Be aware for sign and symptoms of hypoglycemia around 30 minutes to one hour after taking it Lente: Long acting - it will be taken once a day • Take it at night as directed • There is no appreciable peak in this medication - it acts slowly over 24 hours.

How does the nurse evaluate the effectiveness of teaching?

Listen to what the patient is saying to you and determine whether what they are saying is appropriate. If you are teaching a hands on skill - you can watch the patient perform the skill (check their pulse rate).

First dose is higher than following doses

Loading dose

Angiotensin II receptor blockers

Losartan (Cozaar) Valsartan (Diovan) Mechanism of action: Blocks the action of angiotensin II - blood vessels, adrenal glands, and other tissues Indications: no benefit over ACE except no cough Contraindications: renal artery stenosis

When administering cholestyramine (Questran) what nursing interventions are important?

Make sure you give other medications at least one hour before other medications OR four hours after. The medication will bind with other drugs and impair absorption. When binding occurs two things are possible. Least troublesome is that the Questran or other medication may be rendered ineffective. Or worse, Questran will bind with another drug that is life threatening (imagine an antiepileptic drug or an antiviral (HIV) drug not working!

CHF Class 3

Marked limitation of physical activity. Comfortable at rest, but less than ordinary activity causes fatigue, palpitation, or dyspnea.

Pharmacodynamics

Mechanism of action

Neostigmine (Prostigmin)

Mechanism of action Pharmacologic effects Therapeutic administration - muscarinic receptors Neuromuscular effects Therapeutic dose - increases force of contraction in skeletal muscle Toxic levels - decrease force of contraction Central nervous system Therapeutic levels - mild stimulation Toxic levels - depress the CNS

Levodopa/Carbidopa (Sinemet & Paracopa

Mechanism of action Promotion of dopamine synthesis and release Prevention of dopamine degradation Pharmacokinetics: Levodopa converted to dopamine Carbidopa works in gut and tissues to prevent breakdown of levodopa

Levothyroxine

Mechanism of action: Dose gets converted to T3 in the body -- can produce adequate levels of T3 and T4 Pharmacokinetics: One month is required to reach plateau Steady between doses Drugs that reduce levothyroxine absorptions: Cholestyramine (Questran), sucralfate (Carafate) Calcium and Aluminum antacids (Maalox and Mylanta) Drugs that accelerate levothyroxine metabolism: Warfarin (Coumadin) and Catecholamines (dopamine and epinephrine)

Propylthiouracil (PTU) Methimazole (Tapazole)

Mechanism of action: Blocks thyroid hormone synthesis Also acts in periphery to prevent T4 to T3 Pharmacokinetics: However, it does not destroy existing stores of thyroid... May take 3-12 weeks to produce a euthyroid state

Celecoxib (celebrex)

Mechanism of action: Primarily blocks COX II (mediates pain) Less blocking of COXI (mediates gastric lining) Therapeutic use: Patients at risk for gastritis from a non selective NSAID Drug Interactions: Warfarin - increases the anticoagulant effects - increase risk of bleeding

Carbamazepine

Mechanism of action: Suppress high frequent neuronal firing discharge Pharmacokinetics: Hepatic elimination Decreases half life d/t induction of hepatic drug induces metabolism Therapeutic uses: Seizures Bipolar disorder Trigeminal & glossopharyngeal neuralgias Adverse effects: CNS symptoms - nystagmus and ataxia Anemia Leukopenia: less than 3,000 mm3 Thrombocytopenia: less 150,000 - 450,000 Nursing care: Assess for signs of infection Assess for neurological signs and symptoms

Salicylates (Aspirin, ecotrin)

Mechanism of action: Affects COXI more Inhibits thromboxane (which prevents the platelet from working normally) Platelet affects are irreversible

Beta blockers

Mechanism of action: Bind to beta-adrenoceptors located in cardiac nodal tissue, conduction tissue and myocytes Slows the nerve impulses that travel through the heart. As a result, your heart does not have to work as hard because it needs less blood and oxygen. Also block the impulses that can cause arrhythmia's

Valproic acid

Mechanism of action: Therapeutic uses: Seizure disorder Bipolar disorder and migraine Adverse effects: Hepatotoxicity Pancreatitis GI - N&V

Leucovoria

Medication co-administered with methotrexate to preserve folic acid

Glutamergic Inhibitors

Memantine (Namenda) Mechanism of action: reduces high levels of glutamate Nursing care: Monitor respiratory and CV status Assess for and report S&S of focal neurologic deficits (TIA, ataxia, vertigo). Monitor periodic Hct & Hgb, na, Alk Phos, blood glucose. Monitor diabetics for loss of glycemic control.

Saline and Osmotic Laxatives

Milk of magnesia (MOM) Polyethylene glycol (Miralax) Adverse effects: Diarrhea Cramping Hypermagnesemia (MOM)

____ is the only drug approved for prophylaxis of NSAID-induced ulcers.

Misoprostol (Cytotec) ____ is the only drug approved for prophylaxis of NSAID-induced ulcers. Make sure the patient is not pregnant as this would have terrible consequences. Be sure the patient is on birth control. Prostaglandin's are protective of the GI tract. Misoprostol is a prostaglandin analogue. Prostglandins also have a role in ripening the cervix

Leukotriene Modifiers

Montelukast Singulair Mechanism of action: Block leukotrienes Adverse effects: Headache Nasal congestion GI upset

Drug schedule 2

Morphine, methadone, Percet

A patient is status post knee replacement. Which anticoagulant would the nurse expect the medical provider to order?

Most likely enoxparin (Lovenox) subcutaneous or subcutaneous heparin these are used in postoperative patients. Intravenous heparin would not be used because this would increase risk of bleeding post operatively.

Albumin

Most prevalent protein in plasma. MOst important protein to which drugs bind to.

Rhabdomyolosis

Muscle beak down that gets clogged in kidneys

Nystatin

Mycostatin, Nilstat Therapeutic use: Candidal infection of the skin, vagina, esophagus Adverse effects: Topical - local irritation

Polymyxin

Neosporin/ Triple Action Therapeutic use: Gram - organisms P. aeruginosa E. coli, enterbacter, klebsiella Adverse effects: Nephrotoxicity Neurotoxicity Patient management: Total daily dose should not exceed 200 mg day to prevent neurotoxicity

Oprelvekin Thrombocytopenia

Neumega Mechanism of action: stimulates megakaryocytes and thrombopoietin Pharmocokinetcs: onset is 5 - 9 days Subcutaneous only Adverse effects: Fluid retention Nursing care for neumega: Assess for hypersensitivity

Nicotinic Acid

Niacor, Niaspan Action: Indications: Raises HDL levels higher than any other Effective in reducing triglyceride levels. It lowers LDL-cholesterol. It is the least expensive cholesterol drug

Nitrates

Nitroglycerin Method of action: Works mainly by dilating veins Therapeutic use: Relief of acute angina Prophylaxis of angina Treatment of chronic angina

Permithrin

Nix Elimite Adverse reactions: Transient burning, stinging, and pruritis

Naloxone (Narcan)

No analgesic activity at all Competitive antagonist at mu, kappa, and sigma receptor Displaces morphine from all receptors, reversing effects of morphine. onset 3-5 mins duration 30-45 mins

CHF Class1

No limitation of physical activity. Ordinary physical activity does not cause undue fatigue, palpitation, or dyspnea (SOB).

Tamoxifen Antiestrogen

Nolvadex Mechanism of action: Blocks estrogen receptors in breast Stimulates estrogen elsewhere Indications: Prophylaxis Breast cancer that is estrogen receptor positive Nursing care Assess for liver injury and monitor AST, ALT Resumption of menses

A client calls the medical office where the nurse is working and states "I am having chest pain again, I think I am having another heart attack? What advice would you give over the telephone and what would you do?

Note that in this question, the patient indicates that this is a second cardiac event. It is highly likely that they have nitroglycerin at home. Take a nitroglycerin would be the best advice given this circumstance. Having the patient call 911 sounds tempting to do - however that would require the patient to hang up the phone. Assessing if the client is at home alone would be a good second action.

Serotoin receptor Antagonist

Ondansetron (Zofran) - Blocks 3 serotonin receptors

Lispro (Humalog), aspart (Novolog) Rapid Acting

Onset 15 minutes Peak 60 - 90 minutes Duration 3 - 4 hours

Glargine (Lantus) Long acting

Onset: 1-2 hrs Duratioin 24+ hrs DO NOT MIX

NPH Intermediate acting

Onset: 1-2 hrs Peak 4-12 hrs Duration 14-24 hrs

Normal PLT count

PLT count 150,000 - 450,000

Delegation: You are the nurse on a cardiac floor and have just finished receiving report. It is 0800 in the morning and the patients listed below are your patients. In the space below: (1) number the clients in the order in which you would see them after getting report and (2) identify what assessments you would perform.

Patients A 42 year old male that is due 0800 metoprolol (Lopressor) A patient that has slept all night, that is on a stable dose of nitroglycerin IV. A 72 year old that was given nitroglycerin at 0750 by the night shift. A 74 year old warfarin (Coumadin) A 56 year old due for isosorbide dinitrate (Isordil) at 1000. This patient is being discharged. Number the patients in the order in which you would assess them, and identify any relevant nursing responsibilities 3/4. Regular priority 2. Any IV vasoactive medication would be a priority. At the onset of the shift you would need to calculate and verify the dose the patient is receiving. 1. See this patient first! If a patient was given Nitroglycerin they may still be having chest pain. You would always reassess a patient that has received nitroglycerin after 5 minutes. 3/4. Regular priority. Prior to giving any warfarin (Coumadin) be sure that you assess the INR. No warfarin should be given until this lab result comes back. 5. This patient is "last priority" considering this group of patients. When you perform patient teaching make sure that you include the following points: change positions slowly

Bismuth subsalicylate

Pepto Bismol Not often thought of as an antibiotic; Pepto Bismol acts as an antibiotic; disrupting the cell wall of H. pylori Makes tongue black - warn patient Stool discoloration

P.E.A.R. S.A.M.

Physical dependence Euphoria Analgesia Respiratory depression Sedation Analgesia Miosis

Thiazolidinediones

Pioglitazone (Actos), Rosiglitazone (Avandia) Mechanism of action: reduces insulin resistance Activates a receptor - PPAR gamma Insulin must be present for it to work Kinetics: Half life 2 hours longer in liver disease

Penicillin

Piperacillin Amoxicillin Ampicillin

Cisplatin Alkylating Agents

Platinol-AQ Mechanism of action: Forms cross links between DNA strands Nursing care: Monitor for signs of hearing loss Teach patient to report buzzing, ringing in ears Typically see IV hydration and diuretics Highly emetogenic

Describe how the nurse would infuse a potassium infusion?

Potassium should be diluted and infused slowly or cardiac dysrhythmias may result. Potassium should never be directed infused.

Dopamine Agonists

Pramipexole (Mirapex), Ropinirole (Requip) Therapeutic uses: used early in disease - and later in combination with other agents Mechanism of action: Binds selectively to D2 and D3 receptors

On day three of the hospital admission the physician prescribes prednisone for the patient/ Why is this drug prescribed for IBD? And what adverse effects are important to assess for in the acute phase of being treated with this drug?

Prednisone is a steroid that is given during acute exacerbations to induce a quick remission. These drugs are very powerful and will work to stabilize the patient. The draw back is that they cannot be used on a daily basis because of issues of bone loss, increased appetite, weight gain, HTN, edema, slow wound healing, black and blue ecchymoses on skin, skin thinning, GI ulcers, and depression. The hyperglycemia can occur immediately on steroids

HOrmone replacement therapy

Premarin Estrace Therapeutic use: prevention Mechanism of action: Estrogen inhibits osteoclast activity; resulting in decreased bone loss

The nurse is reviewing the mechanism of action of cholinergic drugs. The desired effects of cholinergic drugs come from the stimulation of which receptors? A. Muscarinic B. Nicotinic C. Cholinergic D. Ganglionic

Rationale: A; The desired effects come from muscarinic receptor stimulation; many of the undesirable adverse effects are due to nicotinic receptor stimulation.

A patient arrives at the emergency department and you are the triage nurse. A patient has been taking aspirin for a year. What type of symptoms would be concerning for the nurse to hear from the patient?

Probably the worse would be if a patient stated I have black stools. This indicates bleeding which may be an adverse effect of aspirin.

Aldesleukin Interleukins

Proleukin Mechanism of action: activates T lymphocytes Adverse effect: Interleukin-2 and alpha interferon: may cause severe flulike symptoms, with chills and fever of 103°, NVD, anorexia, fatigue

Tacrolimus

Protopic Pimecrolimus Elidel Mechanism of action: Inhibit T lymphocyte activation Prevent release of inflammatory cytokines and mediators from mast cells Therapeutic use: Short term and intermittent therapy for atopic dermatitis Adverse effects: Erythema, pruritis and burning Burning sensation in the applied area; improves with continued use Black box warning: tumors in animals; reports of lymphoma in humans Nursing care: Implement: do not to apply an occlusive dressing Instruct patient to report and new skin lesions or changes Instruct patient they may feel transient burning

Bulk-forming laxatives

Psyllium (Metamucil) Methycellulose (Citrucel) Adverse effects: Esophageal or GI obstruction

A patient is being discharged to home on a single daily dose of a diuretic. The health care provider instructs the patient to take the dose at which time so it will be least disruptive to the patient's daily routine? A. In the morning B. At noon C. With supper D. At bedtime

Rationale: A; Take the diuretic medication early in the morning to prevent urination during the night. Taking the diuretic at the other times may cause nighttime urination and disrupt sleep.

Liver toxicity

RUQ pain, jaundice, icterus- (yellow eyes)

Selective Estrogen Receptor Modulator

Raloxifene (Evista) Mechanism of action: Estrogenic effects on bone (bone preserving) Antagonistic toward estrogen receptors in the endometrium and breast Contraindication: nursing women or pregnancy Adverse effects: Venous thromboembolism Nursing care:

The nurse is planning care for a client receiving total parenteral nutrition. Which action is most appropriate? A. Maintain a dedicated percutaneous endoscopic gastrostomy (Peg) tube B. Remove the solution 30 minutes from the refrigerator before starting C. Place patient in left lateral recumbent position D. Withhold all oral medications while the patient is receiving TPN

Rationale: A cold infusion could cause irritation to the intravenous (IV) site. Parenteral infusions are done through an intravenous (IV) line, not a percutaneous endoscopic gastrostomy (PEG) tube. Checking the tube for residual is done with enteral feedings, not parenteral feedings. The client can continue to receive oral medications while total parenteral nutrition (TPN) is infusing.

The nurse is preparing to administer the following medications. Which medication should the nurse question administering? A. the regular insulin to a client with blood glucose level of 250 mg / dl. B. metoprolol (Lopressor) to the client with hyperthyroidism. C. dextrose 50% to an the client with hypoglycemia D. 4 ounces of orange juice to a semiconscious client with a blood glucose 55.

Rationale: A is acceptable because insulin in the treatment for hyperglycemia. B is acceptable because beta blockers are often given to clients with hyperthyroidism because of high heart rates. C is acceptable because dextrose would treat hypoglycemia. D is not acceptable because an oral antidote to hypoglycemia would not be given to a client with hypoglycemia.

. The client is diagnosed with hypothyroidism. Which signs / symptoms would the nurse expect the client to exhibit? A. complaints of extreme fatigue and hair loss. B. exophthalamos and complaints of nervousness. C. complaints of profuse sweating and flushed skin. D. tetany and complaints of stiffness of the hands.

Rationale: A is correct because a decrease in thyroid hormone causes decreased metabolism, which leads to fatigue and hair loss. B is incorrect because these are signs of hyperthyroidism. C is incorrect because these are signs of hyperthyroidism. D is incorrect because these are signs of parathyroidism.

A client with a partial occlusion of the left common carotid artery is to be discharged while taking warfarin (Coumadin). When discussing the adverse effects of warfarin (Coumadin), the nurse should tell the client to consult with the physician if A. blood appears in the urine. B. swelling of the ankles appears. C. the ability to concentrate disappears. D. they develop a headache.

Rationale: A is correct because patients taking warfarin (Coumadin) are at increased risk of bleeding. B, C and D are distractor's

TPN is ordered for a patient with Crohn's disease. While administering the TPN solution, it is important for the nurse to remember that TPN is used to: A. increase cell nutrition B. treat metabolic acidosis C. provide hydration D. reverse a positive nitrogen balance

Rationale: A is correct because the goal is to meet the client's metabolic needs. B is not correct because TPN is not used to treat metabolic acidosis; ketoacidosis can actually develop as a result of administering TPN. C is not correct because TPN is a hypertonic solution it is not sued to meet the hydration needs of patients. D is not correct because TPN is used to provide a positive nitrogen balance.

The nurse is preparing to administer an evening dose of regular insulin to a patient who is receiving total parenteral nutrition (TPN). Which action is most appropriate for the nurse to take to determine the amount of insulin to give? A. base the dosage on the glucometer reading of the patients glucose level obtained immediately before administering the insulin B. Base the dosage on the fasting blood glucose level obtained earlier in the day C. Calculate the amount of TPN fluid the patient has received since the last dose of insulin and adjust the dose accordingly D. Assess the patient's dietary intake for the evening meal and snack and adjust the dosage accordingly

Rationale: A is correct because whenever the nurse is using a sliding scale insulin schedule, the nurse should obtain the blood glucose level immediately before giving the insulin. B is not correct because the AM blood glucose is not relevant. C and D are not correct because insulin doses are not calculated based upon TPN intake or dietary intake

93. When counseling a male patient about the possible adverse effects of antihypertensive drugs, the nurse should discuss which potential problem? A. Impotence B. Bradycardia C. Increased libido D. Increased weight

Rationale: A; Sexual dysfunction is a common complication of antihypertensive medications and may be manifested in men as decreased libido or impotence. The other options are not correct.

When counseling a male patient about the possible adverse effects of antihypertensive drugs, the nurse should discuss which potential problem? A. Impotence B. Bradycardia C. Increased libido D. Increased weight

Rationale: A; Sexual dysfunction is a common complication of antihypertensive medications and may be manifested in men as decreased libido or impotence. The other options are not correct.

The nurse is administering lispro insulin and should keep in mind that this insulin will start to have an effect within which time frame? A. 15 minutes B. 1 to 2 hours C. 80 minutes D. 3 to 5 hours

Rationale: A; The onset of action for lispro insulin is 15 minutes. The peak plasma concentration is 1 to 2 hours; the elimination half-life is 80 minutes; and the duration is 3 to 5 hours.

Pioglitazone (Actos) is ordered for a client. What adverse effects should the nurse monitor for? A. congestive heart failure B. myalgia C. sodium retention D. glycosuria.

Rationale: A; this category of medications causes fluid retention which can be deadly in a patient with heart failure because they already retain water.

. A client is being treated for hypothyroidism. Which of the following findings indicate that thyroid replacement therapy has been inadequate? Select all that apply. A. tachycardia B. low body temperature C. nervousness D. bradycardia E. dry mouth

Rationale: B and D are correct. During hypothyroid states, metabolism is slower. A low body temperature and bradycardia indicate that replacement therapy is inadequate. A, C, and E are incorrect because tachycardia, nervousness and dry mouth are symptoms of excessive level of thyroid hormone; thus indicating the client has received an excessive dose of thyroid hormone.

Which of the following interventions should the nurse include in the patients plan of care to prevent complications associated with TPN administration through a central line? A. use clean technique for all dressing changes B. tape all connections in the system C. encourage bed rest D. cover the insertion site with a sterile gauze

Rationale: B is correct because infection and air embolism are complication associated with TPN. A is not correct because strict aseptic technique is used for dressing changes. C is not correct because ambulation and activities are encouraged and not limited during TPN. D is not correct because the dressing should be an air occlusive dressing.

A client is prescribed Aspirin for the prevention of stroke. The client asks the nurse about common adverse effects of Aspirin. The nurse should teach the client about possible A. hypotension. B. gastric distress. C. headache. D. weakness.

Rationale: B is correct because it is a side effect. A is incorrect because aspirin does not cause hypotension. C is incorrect because headache is not a side effect of aspirin. D is incorrect because aspirin does not cause weakness.

A patient has been started on long term sulfasalazine (Azulfidine) for treatment of ulcerative colitis. The nurse should encourage the patient to eat which of the following foods to help avoid the nutrient deficiencies that may develop as a result of this medication? A. citrus fruits B. green, leafy vegetables C. eggs D. milk products

Rationale: B is correct because patients may develop folic acid deficiency. The patient can take folic acid supplements, but the nurse should encourage the patient to increase the intake of folic acid in the diet. These vegetable are high in folic acid. Although A, C and D are "healthy choices" they are not good sources of folic acid; therefore these choices are not good answers.

A nurse is monitoring a client that is receiving aspirin after an MI. Which of the following actions would the nurse implement? A. Evaluate deep tendon reflexes B. Assess for ringing in the ears. . C. Monitor AST/ALT D. Teach a low vitamin K diet

Rationale: B is correct because ringing in the ears indicates salicyclism. A is incorrect because it has nothing to do with aspirin. C is incorrect because aspirin does not cause damage to the liver (hepatotoxicity) and AST/ALT monitor liver disease. D is incorrect because a low vitamin K diet pertains to warfarin (Coumadin).

When assessing for hypoglycemia, the nurse would expect to find: A. lethargy B. tachycardia C. warm, dry skin D. increased respirations

Rationale: B is correct because this is results from sympathetic nervous system activity. A is incorrect because it is a sign of hyperglycemia and is related to metabolic acidosis and inadequate energy production. C is incorrect this is a sign of hyperglycemia, it is caused by the dehydration associated with osmotic diuresis related to glycosuria (glucose in urine). A helpful mnemonic to remember the signs and symptoms of hypoglycemia and hyperglycemia: "cold and clammy = need some candy." "hot and dry = sugar high."

A nurse is teaching a client on warfarin (Coumadin) about dietary restrictions. It would be most appropriate for the nurse to state: A. "you need to make sure you eat a banana each day." B. "you will need to limit your intake of green leafy vegetables." C. "you can't have any aged cheeses." D. "do not eat protein for 2 hour after taking the medication."

Rationale: B is correct because vegetables that are high in vitamin K will antagonize the effects warfarin (Coumadin). A, C and D are distractor's and are appropriate with other medications.

Lomotil (diphenoxylate hydrochloride and atropine sulfate) is prescribed for the patient with ulcerative colitis. The nurse monitors the patient knowing that which of the following is a therapeutic effect of this medication? A. Elimination of peristalsis B. Decreased diarrhea C. Decreased cramping D. Improved intestinal tone

Rationale: B is correct. Lomotil is an antidiarrheal product which decreases the amount of stools by reducing the volume of liquid in stools.

A patient is taking nicotinic acid (Niacin) to decrease cholesterol and triglycerides. In counseling her about taking her medication, you explain that flushing a common adverse effect of niacin therapy can be decreased A. taking half the dose in the morning and half the dose at night B. taking aspirin 325 mg 30 minutes before the niacin C. taking the drug on an empty stomach D. taking niacin as monotherapy for dyslipedemia

Rationale: B is the correct answer.

Orthostatic or postural hypotension is a potential outcome of nitroglycerin therapy that places the elderly at risk for injury from falls. Instructions given to the patient to decrease this effect of antihypertensive therapy include A. take the dose in the early afternoon B. change position slowly when rising from bed or chair C. increase fluid intake by 500 mL per day D. decrease the dose until symptoms disappear

Rationale: B is the correct answer. A is incorrect because it is better to take the medication at nighttime when the patient will remain lying for several hours. C will not prevent orthostatic or postural hypotension. D is not the correct answer as it is not good advice to recommend patients change their doses.

A cholinergic drug is prescribed for a patient. The nurse checks the patient's medical history, knowing that this drug is contraindicated in which disorders? Select all that apply. A. Bladder atony B. Gastrointestinal obstruction C. Bradycardia D. Alzheimer's disease E. Hypotension F. Chronic obstructive pulmonary disease

Rationale: B, C, E, F; Contraindications to the use of cholinergic drugs include gastrointestinal or genitourinary obstruction, bradycardia, hypotension, and chronic obstructive pulmonary disease. The other options are possible indications for cholinergic drugs.

During diuretic therapy, the nurse monitors the fluid and electrolyte status of the patient. Which are symptoms of hyponatremia? Select all that apply. A. Red, flushed skin B. Lethargy C. Decreased urination D. Hypotension E. Stomach cramps F. Elevated temperature

Rationale: B, D, E; Hyponatremia is manifested by lethargy, hypotension, stomach cramps, vomiting, diarrhea, and seizures. The other options are symptoms of hypernatremia.

The nurse is developing a plan of care for a patient with Crohn's disease who is receiving total parenteral nutrition (TPN). Which of the following interventions should the nurse include? Select all that apply. A. monitoring the vital signs once a shift B. weighing the patient daily C. changing the central line dressing daily D. monitoring the IV infusion hourly E. taping all connections securely

Rationale: B, D, and E are correct. A is incorrect because vital signs should be assessed every 4 hours to detect complications initially. C is incorrect because it is recommended that the dressing is changed once to twice a week or when it becomes soiled, loose or wet.

During a teaching session for a patient on antithyroid drugs, the nurse should discuss which dietary instructions? A. Using iodized salt when cooking B. Avoiding foods containing iodine C. Increasing fluid intake to 2500 mL per day D. Increasing intake of sodium- and potassium-containing foods

Rationale: B; Patient's on antithyroid therapy should avoid iodine-containing foods. These foods may interfere with the effectiveness of the antithyroid drug. The other options are not correct.

A patient who is taking propylthiouracil for hyperthyroidism wants to know how this medicine works. What should the nurse explain to her? A. It blocks the action of thyroid hormone. B. It impedes the formation of thyroid hormone. C. It destroys overactive cells in the thyroid gland. D. It inactivates already existing thyroid hormone in the bloodstream.

Rationale: B; Propylthiouracil impedes the formation of thyroid hormone but has no effect on already existing thyroid hormone.

When monitoring a patient who has been on peripheral parenteral nutrition (PPN) for more than 3 weeks, the nurse monitors for which possible complication? A. Diarrhea B. Phlebitis C. Hypokalemia D. Hypoglycemia

Rationale: B; The long-term administration of nutrition supplements via a peripheral vein may lead to phlebitis and, possibly, the loss of a limb.

The physician prescribes sulfasalazine (Azulfidine) for a patient with ulcerative colitis to continue taking at home. Which instruction should the nurse give the patient about taking this medication? A. avoid taking it with food B. take the total dose at bedtime C. take it with a full glass of water D. stop taking it if urine turns orange-yellow

Rationale: C is correct because adequate fluid intake of at least 8 glasses a day to prevent crystalluria and stone formation during treatment. A and B are incorrect because it is best taken with meals and in equally divided doses. D is incorrect because this medication gives alkaline urine an orange-yellow color but it is not necessary to stop taking the medication when this occurs.

A client who is taking an oral hypoglycemia daily for type 2 diabetes develops the flu and is concerned about the need for special care. The nurse should advise the client to: A. skip the oral hypoglycemia pill, drink plenty of fluids and stay in bed. B. avoid food, drink clear fluids, take a daily temperature and stay in bed. C. take the oral hypoglycemic pill, drink warm fluids, and perform a serum glucose test before meals. D. eat as much as possible, increase fluid intake, and call the office the next day.

Rationale: C is correct because physiological stress increases gluconeogenesis, requiring continued to control glucose level; fluids prevent dehydration; monitoring glucose levels permits early intervention if hospitalization is necessary. A is incorrect because skipping the oral hypoglycemic could precipitate hyperglycemia. B is incorrect because food intake should be attempted to prevent acidosis. D is incorrect; because eating as much food as possible may precipitate hyperglycemia; these are also incomplete instructions, oral hypoglycemia agents should be taken.

The client diagnosed with hypothyroidism is prescribed levothyroxine (Synthroid). Which assessment indicates the medication has been effective? A. the client has a three pound weight gain. B. the client has a decreased pulse rate. C. the client's temperature is within normal limits. D. the client denies diaphoresis.

Rationale: C is correct because the client with hypothyroidism frequently has subnormal temperature, so a temperature within normal limits indicates the medication is effective. A is not correct because levothyroxine will help decrease the client metabolic rate. A weight gain indicates that the thyroid replacement is not high enough. B is not correct because a decreased pulse rate indicates that there is not enough thyroid hormone level; therefore the medication is not effective. D is not correct because diaphoresis occurs with hyperthyroidism.

The nurse is assessing a client for risk factors of heart disease. Which of the following factors is considered a risk factor for disease? A. HDL 72 mg/dL B. aged of 37 years C. LDL 172 mg/dL D. Total cholesterol 152 mg/dL

Rationale: C is correct, an elevated LDL directly correlates to heart disease. A is incorrect because an elevated HDL is protective against heart disease. B is incorrect because young age is protective against heart disease. D is incorrect because this is within normal limits.

Which signs or symptoms would indicate that the client with hypothyroidism is not taking enough thyroid hormone? A. complaints of weight loss and fine tremors. B. complaints of excessive thirst and urination. C. complaints of constipation and being cold. D. complaints of delayed wound healing and belching.

Rationale: C is correct, since these are signs of hypothyroidism. A is incorrect because weight loss and fine tremors would make the nurse suspect the client is taking too much thyroid hormone because these are symptoms of hyperthyroidism. B is incorrect because excessive thirst and urination are symptoms. D is incorrect.

The client received 10 units Humulin R at 0700. At 1030 the CNA tell the nurse the client has a headache and is acting "funny." Which action should the nurse implement first? A. instruct the assistant to obtain blood glucose first. B. have the client drink eight ounces of orange juice. C. go to the client's room and assess the client for hypoglycemia. D. prepare to administer one amp of 50% dextrose intravenously.

Rationale: C is correct. A is incorrect because the blood glucose level should be obtained but it is not the first intervention. B would be correct once the patient is found to be hypoglycemia. D 50% is only administered if the client is unconscious and the nurse suspects hypoglycemia.

A client is taking tissue plasminogen activator (tPA). It is important for the nurse to do all of the following except A. Monitor neurological status B. Monitor vital signs C. Keep antidote (vitamin K) available D. Assess PT and INR

Rationale: C is the answer because Vitamin K is the antidote for warfarin (Coumadin); not tPA. A is important because it is important to monitor neurological status because this medication increases risk of bleeding in all areas! Bleeding in the brain is dangerous and fatal. B is correct because changes in vital signs may indicate bleeding - specifically tachycardia is a sign of bleeding. D is incorrect because Pt and INR are used to monitor warfarin (Coumadin).

A patient has been taking aluminum hydroxide (Amphogel) 30 mL 4 times a day. The patient tells the nurse he has been unable to have a bowel movement for 3 days. Based on this information, the nurse would determine that which of the following is the most likely cause of the patient's constipation? A. the patient has not been taking enough fiber in his diet B. the patient needs to increase his daily exercise C. the patient is experiencing an adverse effect of the aluminum hydroxide D. the patient has developed a gastrointestinal obstruction

Rationale: C is the correct answer - it is most likely the patient is experiencing an adverse effect of the antacid. Antacids with aluminum hydroxide form insoluble salts in the body which then precipitate and accumulates in the intestines causing constipation. D is not correct because constipation in the absence of other symptoms is not likely a sign of bowel obstruction.

A patient has been taking magnesium hydroxide (milk of magnesia) at home to control Hiatal hernia symptoms. The nurse should assess the client for which of the following conditions most commonly associated with the ongoing use of magnesium based antacids? A. anorexia B. weight gain C. diarrhea D. constipation

Rationale: C is the correct answer as the magnesium slats in magnesium hydroxide are related to those found in laxatives and may cause diarrhea. Aluminum salt products can cause constipation. Many patients find that a combination product is required to maintain normal bowel function. The use of magnesium hydroxide does not cause anorexia or weight gain.

Famotidine (Pepcid) is ordered for a patient with hiatal hernia. This medication is used to prevent which of the following? A. esophageal reflux B. Dysphagia C. Esophagitis D. Ulcer formation

Rationale: C is the correct answer. This medication is a histamine receptor antagonist that decreased the quantity of gastric secretions. It may be used I Hiatal hernia treatment to prevent or treat the Esophagitis and heartburn associated with reflux. This medication is not used to prevent reflux, dysphagia or ulcer development.

When monitoring a patient's response to oral antidiabetic drugs, the nurse knows that the laboratory results that would indicate a therapeutic response would be a(n) A. random blood glucose level above 170 mg/dL. B. blood glucose level of less than 50 mg/dL after meals. C. fasting blood glucose level between 70 and 100 mg/dL. D. evening blood glucose level below 70 mg/dL.

Rationale: C; A fasting blood glucose level between 70 and 100 mg/dL indicates a therapeutic response to glucose-elevating drugs. The other options are incorrect.

When teaching about hypoglycemia, the nurse should make sure that the patient is aware of the early signs of hypoglycemia, including A. hypothermia and seizures. B. nausea and diarrhea. C. irritability and confusion. D. fruity, acetone odor to the breath.

Rationale: C; Early symptoms of hypoglycemia include the central nervous system (CNS) manifestations of confusion, irritability, tremor, and sweating. The other options are incorrect.

The nurse is reviewing instructions for a patient with type 2 diabetes who also gives herself insulin injections as part of the therapy. The nurse asks the patient, "What should you do if your fasting blood glucose is 42 mg/dL?" Which response by the patient is correct? "I will call my doctor right away." "I will give myself the Regular insulin." "I will take an oral form of glucose." "I will rest until the symptoms pass."

Rationale: C; Hypoglycemia can be reversed if the patient eats glucose tablets or gel, corn syrup, or honey; or drinks fruit juice or a nondiet soft drink or other quick sources of glucose, which should always be kept at hand. She should not wait for instructions from her physician nor delay taking the glucose by resting. The Regular insulin would only lower her blood glucose levels more.

A patient who has had abdominal surgery has been discharged on a cholinergic drug to assist in increasing gastrointestinal peristalsis. The nurse should teach this patient to look for which therapeutic effects? A. Decreased pulse rate B. Abdominal cramping C. Increased bowel sounds and passage of flatus D. Decreased frequency and urgency of voiding patterns

Rationale: C; In patients suffering a decrease in gastrointestinal peristalsis postoperatively, taking a cholinergic drug should result in an increase in bowel sounds, the passage of flatus, and the occurrence of bowel movements that indicate increased gastrointestinal peristalsis.

When reviewing the health history of a patient, the nurse recalls that potassium supplements are contraindicated in patients with a history of which problem? A. Burns B. Diarrhea C. End stage renal disease D. Use of loop diuretics

Rationale: C; Potassium supplements are contraindicated in the presence of renal disease because renal disease causes hyperkalemia.

Levothyroxine has been prescribed for a patient with hypothyroidism. The nurse provides information to the patient about the medication and tells the patient to contact the prescriber if which potential adverse effect occurs? A. Fatigue B. Constipation C. Palpitations D. Drowsiness

Rationale: C; Some of the more serious adverse effects of the thyroid drugs include tachycardia, palpitations, and chest pains. The other options are not adverse effects of thyroid replacement drugs.

When reviewing the mechanisms of action of diuretics, the nurse knows that which statement is true about the furosemide (Lasix)? A. They work by inhibiting aldosterone. B. They are very potent, having a diuretic effect that lasts at least 6 hours. C. They are particularly useful when rapid diuresis is desired because their onset of action is rapid. D. They have the disadvantage of ceasing to be effective when the creatinine clearance decreases below 25 mL/min.

Rationale: C; The loop diuretics have a rapid onset of action; therefore, they are useful when rapid onset is desired. Their effect lasts for about 2 hours, and a distinct advantage they have over thiazide diuretics is that their diuretic action continues even when creatinine clearance decreases below 25 mL/min

When reviewing the mechanisms of action of diuretics, the nurse knows that which statement is true about the loop diuretics? A. They work by inhibiting aldosterone. B. They are very potent, having a diuretic effect that lasts at least 6 hours. C. They are particularly useful when rapid diuresis is desired because their onset of action is rapid. D. They have the disadvantage of ceasing to be effective when the creatinine clearance decreases below 25 mL/min.

Rationale: C; The loop diuretics have a rapid onset of action; therefore, they are useful when rapid onset is desired. Their effect lasts for about 2 hours, and a distinct advantage they have over thiazide diuretics is that their diuretic action continues even when creatinine clearance decreases below 25 mL/min

When a patient is receiving diuretic therapy, which of the following would best reflect the patient's fluid volume status? A. Blood pressure and pulse B. Serum potassium and sodium levels C. Intake, output, and daily weight D. Measurements of abdominal girth and calf circumference

Rationale: C; Urinary intake and output and daily weights are the best reflections of a patient's fluid volume status.

The nurse is changing the subclavian dressing of a patient who is receiving total parenteral nutrition. When assessing the catheter insertion site, the nurse notes the presence of yellow drainage from around the sutures that are anchoring the catheter. Which action should the nurse take first? A. clean the insertion site and redress the area B. document the assessment findings in the patients chart C. obtain a culture specimen of the drainage D. assess the patients capillary blood glucose

Rationale: C; central lines / access sites are at high risk of infection and yellow drainage indicates possible infection.

A patient who has type 2 diabetes is receiving metformin (Glucophage) is scheduled for a CT scan in the AM. What is the best action regarding the administration of her oral antidiabetic drugs? Giving her half the original dose Withholding all medications as ordered Contacting the prescriber for further orders Giving her the medication with a sip of water

Rationale: C; the prescriber should be contacted for further orders regarding the administration of the oral antidiabetic drugs. The other options are not correct.

A physician prescribes the potassium sparing diuretic spironalactone (Aldactone). In which of the following conditions should the nurse discontinue the drug? A. Gout attacks B. Elevated blood glucose C. Serum potassium level greater 5.2 mEq/mL D. Migraine headaches

Rationale: C; this medication causes hyperkalemia

A patients TPN fluid is being administered through a central IV tubing. Which complication can occur if the tubing becomes disconnected? A. phlebitis B. pneumothorax C. hemorrhage D. Air embolus

Rationale: D is correct because air embolus can occur if the tubing is disconnected.

. A client with type 2 diabetes is taking one oral tablet daily. The client asks whether he can skip the pill if he exercises. The nurse should reply: A.. "You will need to decrease your exercise." B. "An extra pill will help your body use glucose correctly." C. "your diet and medicine will not be affected by exercise." D. "No, but you should observe for signs of hypoglycemia while exercising."

Rationale: D is correct because exercise improves glucose metabolism with exercise there is a risk of hypoglycemia, not hyperglycemia. A is incorrect because exercise should not be decreased because it improves glucose metabolism. B is incorrect because exercise alone improves glucose metabolism. C is incorrect because control of glucose metabolism is achieved through a balance of diet, exercise and pharmacological therapy.

The nurse would regularly assess a patient's ability to metabolize the TPN solution adequately by monitoring the patient for which of the following signs? A. Tachycardia B. Hypertension C. Elevated BUN D. Hyperglycemia

Rationale: D is correct because the patient should be monitored regularly for hyperglycemia. The client may require small amounts of insulin to improve glucose metabolism. The client should also be monitored for hypoglycemia which may occur if the body overproduces insulin in response to a high glucose intake or if too much insulin is administered. A and B are not correct because they do not indicate the patient's ability to metabolize the solution. C is not correct because an elevated BUN is indicative of renal status and fluid balance.

A client is receiving IV Heparin and oral warfarin (Coumadin) concurrently for a partial occlusion of the left common carotid artery. The client expressed concern about why both Heparin and warfarin (Coumadin) are needed. The nurses explanation is based upon the knowledge that A. this regimen allows clot dissolution and prevents new clot formation. B. this plan permits the administration of smaller doses of each medication. C. both Heparin and warfarin (Coumadin) immediately protect against clot formation. D. anticoagulant effects are provided intravenously until the oral drug reaches it therapeutic level.

Rationale: D is correct. A is incorrect because this category of medication does not dissolve clots. B is incorrect. C is incorrect because warfarin (Coumadin) does not immediately protect against clot formation. If you picked C it might be because you read the responses quickly. It is true, heparin injections will immediately offer protection against clot formation.

A nurse is teaching their patient about sulfasalazine (Azulfidine). Which statement made by the patient indicates a need for further instruction? A. "Sensitivity to sunlight may occur" B. "I need to take the medication with meals" C. "This medication should be taken as prescribed" D. "The medication will cause constipation"

Rationale: D is the answer because constipation is not associated with this medication. Azulfidine is an anti-inflammatory sulfonamide. A is not correct because it can cause photosensitivity and the client should be instructed to avoid sun and ultraviolet light. B is not correct because the medication should be administered with food to prolong intestinal passage. C is not correct because the client needs to take the medication as prescribed and continue for the full length of treatment even if symptoms are relieved.

While monitoring a patient who had surgery under general anesthesia 2 hours ago, the nurse notes a sudden elevation in body temperature. This finding may be an indication of which problem? A. Tachyphylaxis B. Postoperative infection C. Malignant hypertension D. Malignant hyperthermia

Rationale: D; A sudden elevation in body temperature during the postoperative period may indicate the occurrence of malignant hyperthermia, a life-threatening emergency. The elevated temperature does not reflect the other problems listed.

A patient has been receiving total parenteral nutrition (TPN). Upon assessment, the nurse notes that his blood pressure is elevated, his pulse is weak and elevated, he seems confused, and he has new pitting edema around his ankles. Which condition does the nurse suspect the patient is experiencing? A. Hyperglycemia B. Hypoglycemia C. Infection D. Fluid overload

Rationale: D; Fluid overload may also occur with parenteral nutrition. It is manifested by weak pulse, hypertension, tachycardia, confusion, decreased urine output, and pitting edema.

The nurse is teaching a group of patients about self-administration of insulin. What teaching is important to include? A. Patients should use the injection site that is the most accessible. B. During times of illness, patients should increase their insulin dosage by 25%. C. When mixing insulins, the cloudy (such as NPH) insulin should be drawn up into the syringe first. D. When mixing insulins, the clear (such as Regular) insulin should be drawn up into the syringe first.

Rationale: D; If mixing insulins in one syringe, the clear (Regular) insulin should always be drawn up in to the syringe first. Patients should always rotate injection sites and should notify their physician if they become ill. Patients should never adjust their own insulin doses!

During a fishing trip, a patient pierced his finger with a large fishhook. He is now in the emergency department to have it removed. The nurse anticipates which type of anesthesia will be used for this procedure? A. Topical benzocaine spray on the area B. Spinal anesthesia with mepivacaine C. Topical prilocaine (EMLA) cream around the site D. Infiltration of the puncture wound with lidocaine

Rationale: D; Infiltration anesthesia is commonly used for minor surgical procedures. It involves injecting the local anesthetic solution intradermally, subcutaneously, or submucosally across the path of nerves supplying the area to be anesthetized. The local anesthetic may be administered in a circular pattern around the operative field. The others types are not appropriate for this injury.

When monitoring a patient for signs of hypokalemia, the nurse looks for what early sign? Seizures Cardiac dysrhythmias Stomach cramps Muscle weakness

Rationale: D; Muscle weakness is an early symptom of hypokalemia, as are hypotension, lethargy, mental confusion, and nausea. Cardiac dysrhythmias are a late symptom of hypokalemia. The other options are not correct.

When administering a neuromuscular blocking drug such as pancuronium, the nurse needs to remember which principle? A. It can be used instead of general anesthesia during surgery. B. Only skeletal muscles are paralyzed; respiratory muscles remain functional. C. It causes sedation and pain relief while allowing for lower doses of anesthetics. D. Patients will require artificial mechanical ventilation because of paralyzed respiratory muscles.

Rationale: D; Patients receiving neuromuscular blocking drugs require artificial mechanical ventilation because of the resultant paralysis of the respiratory muscles. In addition, they do not cause sedation or pain relief. They are used along with, not instead of, general anesthesia during surgery

A patient has had an overdose of an intravenous cholinergic drug. The nurse should expect to administer which drug as an antidote? A. atenolol B. bethanechol C. dobutamine D. atropine sulfate

Rationale: D; Prompt administration of atropine sulfate can reverse a toxic dose of cholinergic drugs. The other drugs listed are not antidotes to cholinergic toxicity.

During the night shift, a patient's total parenteral nutrition (TPN) infusion ran out, and there was no TPN solution on hand to continue the infusion. The nurse will have to implement measures to prevent what consequence of abruptly discontinuing TPN infusions? A. Fluid overload B. Hyperglycemia C. Dumping syndrome D. Rebound hypoglycemia

Rationale: D; Rebound hypoglycemia may occur if TPN is discontinued abruptly. It may be prevented by infusion of 10% glucose in situations in which TPN must be stopped abruptly

When assessing patients in the preoperative area, the nurse knows that which patient is at a higher risk for an altered response to anesthesia? A. The 30-year-old who has never had surgery before B. The 45-year-old who stopped smoking 10 years ago C. The 20-year-old who is to have a lymph node removed D. The 78-year-old who is to have gallbladder removal

Rationale: D; The elderly patient is more affected by anesthesia than the young or middle-aged adult patient because of the effects of aging on the hepatic, cardiac, respiratory, and renal systems.

A patient is undergoing abdominal surgery and has been anesthetized for 3 hours. Which of the following nursing diagnoses would be appropriate for this patient? A. Anxiety related to the use of an anesthetic B. Risk for injury related to increased sensorium from general anesthesia C. Decreased cardiac output related to systemic effects of local anesthesia D. Impaired gas exchange related to central nervous system depression produced by general anesthesia

Rationale: D; The nursing diagnosis of impaired gas exchange is appropriately worded for this patient. Anxiety would not be appropriate while the patient is in surgery. Sensorium would be decreased during surgery, not increased. Cardiac output is affected by general anesthesia, not local anesthesia.

A patient with primary hypertension is prescribed drug therapy for the first time. The patient asks how long drug therapy will be needed. Which answer by the nurse is the most correct response? "This therapy should take about 3 months." "This therapy should take about a year." "This therapy should go on until your symptoms disappear." "Therapy for high blood pressure is usually life-long."

Rationale: D; There is no cure for the disease, and treatment will be life-long. The other answers are not appropriate.

A patient with primary hypertension is prescribed drug therapy for the first time. The patient asks how long drug therapy will be needed. Which answer by the nurse is the most correct response? A. "This therapy should take about 3 months." B. "This therapy should take about a year." C. "This therapy should go on until your symptoms disappear." D. "Therapy for high blood pressure is usually life-long."

Rationale: D; There is no cure for the disease, and treatment will be life-long. The other answers are not appropriate.

The nurse is preparing to administer medications to a patient who is receiving enteral feeding via a gastrostomy tube. When reviewing the patient's medication list, the nurse notes a potential concern about a food-drug interaction if which medication is listed? A. acetaminophen (Tylenol) B. lorazepam (Ativan) C. metoclopramide (Reglan) D. phenytoin (Dilantin)

Rationale: D; Tube feedings can reduce the absorption of phenytoin, which may result in seizures. It is recommended that tube feedings be held for at least 2 hours before and after the administration of phenytoin.

The nurse is preparing to administer intravenous potassium. Which administration technique is correct? A. The intravenous rate should not exceed 30 mEq/hr. B. Oral forms should be given on an empty stomach to maximize absorption. C. Intravenous solutions should not contain more than 60 mEq/L of potassium. D. When given intravenously, potassium must always be given in diluted form.

Rationale: D; When giving intravenous potassium, the medication must always be given in a diluted form and administered slowly. Intravenous bolus or undiluted forms may cause cardiac arrest. Intravenous rates are not to exceed 20 mEq/hr. Oral forms should be mixed with juice or water or taken according to instructions because the patient usually complains of abdominal pain because it is irritating to the stomach.

A patient taking warfarin (Coumadin) is status post aortic valve replacement. Which of the following data indicates that the medication should be held? A. INR 2.8 B. protime 58 seconds C. INR 6.0 D. protime 35 seconds

Rationale: INR's are used to monitor warfarin (Coumadin). The key word here is "held." An INR of 6 is obviously too high. B and D are distractors.

nurse is counseling a patient about his medication and when he will need his laboratory testing done. What would be your response to him?

Rationale: The patient will need to have lab work checked in 4-6 weeks.

A 36 year old female has been taking lovastatin (Lipitor) for 6 months to treat dyslipedemia. At the clinic appointment the patient tells the nurse she is 6 weeks pregnant. The nurse understands that it is most likely A. the patient will decrease the dose of prenatal vitamins B. to reduce the dosage of her lovastatin (Lipitor) C. to increase the dosage D. about discontinuing the drug during pregnancy

Rationale: This category of medications (statin) should not be taken by pregnant women.

Meglitinides

Repaglinide (Prandin) Mechanism of action: Stimulate insulin release Adverse effects Hypoglycemia Liver disease - impairs metabolism

Retinoic Acids

Retin A Adapalene Differin Mechanism of action: Stimulate epithelial cell turnover (make epithilieal skin turn over faster) Exhibit anti-inflammatory properties Therapeutic use: First line treatment for acne

Methotrexate Antimetabolites

Rheumatrex)\ Mechanism of action: Cell cycle specific - S phase Inhibits folic acid - hence DNA synthesis cytosine arabinoside (Arc-C) cytarabine (Cytosar) and other -ine drugs Mechanism of action: suppresses DNA synthesis Peripheral neurotoxicity

Beta blockers

Role: protect against sympathetic nervous system stimulation Patient needs to have enough volume Carvedilol (Coreg) Metoprolol (Toprol XL)

Diuretics

Role: volume overload Thiazide (HCTZ) - not typically used due to modest diuresis Loop (furosemide/Lasix) - profound diuresis Preferred with decreased cardiac output Used with sparing spironalactone (Aldactone)

Protease inhibitors

Saquinavir (Invirase) Atazanavir (Reyataz) - less side effects Darunavir (Prezista) Mechanism of action: Interferes with protease enzyme - necessary for viral replication. Drug interactions: inhibition of cytochrome P450 enzymes

Antihistamines

Sedating: Diphenhydramine (Benadryl) Non-sedating: cetirazine (Zyrtec); fexofenadine (Allegra) Mechanism of action: Block histamine response - allergy sufferers Anticholinergic effects - prevent mucus secretion

MAO-B Inhibitor

Selegine (Eldepryl, Carbex) Therapeutic uses: Neuroprotective Mechanism of action: MAO-B is the enzyme inactivates dopamine in striatum Preserves dopamine by inhibiting the enzyme that breaks it down Adverse effects: Principle effect is insomnia Nursing care: Improvement in early PD Improvement in "On-Off" and "Wearing Off"

Corticosteriods

Short acting: Hydrocortisone (Cortef) Intermediate acting: Methylprednisolone (Solu-Medrol) and prednisone (Deltasone) Long acting: dexamethasone (Decadron) Therapeutic use: induce remission Adverse effects: Sodium and water retention Muscle wasting (high doses of steroids) Drug interactions: Diuretics NSAIDs and Aspirin

Beta Agonist

Short: Albuterol Proventil Long: Salmeterol Serevent Mechanism of action: Sympathomimetic B2 receptors Activation of B2 receptors bronchodilation Also suppresses histamine release Therapeutic uses: Asthma: use of short acting PRN for breakthrough Or use long acting daily. Also prophylactic use prior to exercise COPD: Relieve symptoms of COPD Inhaled beta2 agonists are tx of choice

Opiods

Shut down brain signals

What are the similarities and differences between low molecular weight heparins like enoxaparin (Lovenox) and heparin?

Similarity: LMWH and Heparin exert their major anticoagulant effects by binding to and activating antithrombin. This is the main similarity between enoxaparin (Lovenox) and heparin. Both are used for prevention of DVT and treatment of DVT and PE. Differences: The elimination half life is longer for LMWH which means more stable dosing. LMWH also bind less to platelets and platelet factor which reduces the risk of bleeding episodes and thrombocytopenia. The effects of protein binding cannot be stressed enough! Because there is less binding - LMWH require no lab monitoring. In contrast, Heparin requires frequent lab monitoring of PTT's to make sure the effects are beneficial and also to make sure the patient is not becoming toxic.

DDP Inhibitors

Sitagliptin (Januvia) and saxagliptin (Onglyza) Mechanism of action: Inactivate the breakdown of incretin mimetics Adverse effects: URI, HA (common) Pancreatitis Nursing care: Assess for pancreatitis

CHF Class 2

Slight limitation of physical activity. Comfortable at rest, but ordinary physical activity results in fatigue, palpitation, or dyspnea.

Potassium Sparing Diuretics

Spironalactone (Aldactone) Triamterene (Dyrenium) Amiloride (Midamor) Mechanism of action: Modest diuresis Blocks the action of aldosterone in the distal nephron Drug interactions: Increased risk of toxicity if given with digoxin Risk of hyperkalemia if given with potassium Nursing Care: Restrict intake of potassium rich foods Do not administer potassium supplements

Match the following symptoms and signs with: stable, unstable and variant angina?

Statements related to angina Type of angina Pain occurs at a predictable time with exercise Stable Treated with metoprolol (Lopressor) Stable, unstable Chest pain disappear with rest within a few minutes Stable Nitrates are useful to treat Stable, unstable Often is associated with diaphoresis Unstable Treated with calcium channel blockers like cardizem Variant Rationale: It is important for the nurse to recognize signs and symptoms of angina because the nurse needs to know how to respond and use nitroglycerin tablets.

Of all the antihyperlipedemic drugs, what adverse effect is really in common among all of them?

Statin drugs, nicotinic acid, and fibrates can all cause muscle damage (myopathy) and elevated liver enzymes / disease. It is important to assess for alcohol history because those clients are more at risk. Be sure to ask the patient about any muscle pain when you assess them.

Thrombolytics

Steptokinase, Alteplase, Urokinase Mechanism of Action: indirectly binds - converts other plasminogen molecules into plasmin (an enzyme which digests fibrin in clots). Indications: Adverse effects: Bleeding Hypotension Antibody

Pharmakenetics

Study of the movement of drug throughout the body

Regular short acting

Subcutaneous, intravenous Onset 30 minutes - 1 hour Peak 2 - 3 hours Duration 3 - 6 hours

nurse is preparing to administer clopidogrel (Plavix) and the patient asks why are you giving me that medicine? What would you say to the patient?

Teach the patient that the medicine works to prevent platelets from clotting. NCLEX Hint: chances are the patient wont know what platelet aggregation is! A common misconception is that students believe clopidogrel (Plavix) works to prevent deep vein thrombosis (DVT). Anticoagulants work in the venous system primarily which is where DVT's form.

Sulfonamides

Sulfamethoxaole Trimethoprim Bactrim Septra Sulfisoxazole Gantrisin Mechanism of action: Suppress bacterial growth by inhibiting folic acid

5-Aminosalicylates

Sulfasalazine (Azulfidine) Mesalamine (Asacol, Pentasa) Mechanism of action: - reduces inflammation Via prostaglandin synthesis Sulfapyridine - adverse effects Therapeutic use: Most effective use against acute UC, Crohn's

Triptans

Sumatriptan (Imitrex) Frovatriptan (Frova) Mechanism of action: Reduces release of inflammatory neuropeptides Decreases vascular inflammation and causes vasoconstriction Contraindications: Not for pregnant women Uncontrolled HTN Family history of CAD or heart attacks Risk factors for coronary artery disease History of stroke Uncontrolled diabetes High cholesterol

Antiepileptic Drugs

Suppress neuronal discharge at the seizures focus and suppress propagation of seizure activity from the focus to other areas of the brain Mechanism of action: Suppression of sodium influx Suppression of calcium influx Potentiation of GABA

Blood pressure classification

Systolic Diastolic Normal <120 <80 PreHTN 120-139 80-89 Stage I 140-159 90 Stage II 160+ Less / = 100 Classification of HTN: Primary Secondary

The most serious toxicity associated with the use of propylthiouracil is ______.

The most serious toxicity associated with the use of propylthiouracil is _agranulocytosis_____.

A patient with an allergy to _________ should be cautious and talk to provider before taking furosemide (Lasix) or sulfonylureas (glyburide or glipizide).

The sulfonamide (sulfa-) structure is shared by many drug classes. In general, past allergy history is pivotal in determining risk of allergic reaction. In some cases, the liklihood of cross allergenicity is low (as in the diuretics classes) so the general advice is to proceed with caution. • Sulfonamide derivative: diuretics, furosemide, hydrochlorothiazide • Sulfonyurea antidiabetic agents: glipizide, glimepiride • Celecoxib (Celebrex): more theoretical risk • Sumatriptan (Imitrex): more theoretic risk • Sulfonamide derivatives: certain antibiotics

A patient with a lack of aldosterone is most likely to develop fluid volume excess or fluid volume deficit? Which category of medications acts by affecting the action of aldosterone?

They are likely to develop fluid volume excess. Aldosterone is a hormone that results in more sodium staying in the body and causes fluid retention. Where salt goes (not the collecting duct), water follows; which increases the volume of urine. Diuretic medications that affect aldosterone block the action of aldosterone include Spironolactone (Aldactone).

The nurse is preparing to administer warfarin (Coumadin). The laboratory results are as follows: PT 40, INR 6.5. Would you administer the drug? Why or why not? What should be done?

This drug should not be given. An INR of 5.5 is too high and the patient would be over coagulated. The patient should be treated with an anticoagulant - vitamin K because they are at high risk for bleeding.

Which of the following data indicates a therapeutic outcome of warfarin (Coumadin) A. PTT 80 seconds B. INR 3 C. Hemoglobin 14 g /dL D. Hematocrit 37%

This question is aimed at therapeutic outcome - the INR is monitored to make sure it is within a certain range. The INR is selected by the physician and it depends on the patients risk for clotting. A is incorrect because PTT's are used to specifically monitor heparin. C is incorrect because although hemoglobin may be useful in monitoring for occult bleeding - it will not tell the nurse whether the therapy is effective. D is incorrect and irrelevant.

What is the difference between thrombolytics such as tissue plasminogen activator (tPA), altepase (Activase) and heparin?

Thrombolytics are the only category of medications that dissolve existing clots. This makes them useful for patients with MI and embolic CVA. In contrast, Heparin prevents new clots from forming. These drugs can be used together - it increases risk of bleeding though.

ADP antagonists

Ticlopidine (Ticlid), Clopidrogrel (Plavix) Mechanism of Action: blocks receptors for ADP. The effects are irreversible Indications: Prevention of ischemic stroke

What are two side effects of niacin therapy?

Tingling and severe flushing happen commonly with niacin therapy. Therefore, it is important to instruct the patient about this.

Salicyclism

Tinnitus, tachypnea, tachycardia, and elevated temperature

What are some important nursing interventions with patients that are receiving antiemetics?

To assure maximal effect from an antiemetic - it is important for the nurse to assess the patient and administer medication first - before the nausea starts. Once a person starts to vomit it is difficult to stop. In some drugs the meds are so emetogenic (highly likely to cause vomiting) that they will be given zofran and steroids together to potentiate each other.

Class 1 (weakest)

Topical corticosteriod classes Hydrocortisone, prednisolone

Class 2

Topical corticosteriod classes Methylprednisolone aceponate, triamcinolone

Open Angle Glaucoma

Trabecular network gets clogged. Overtime IOP increases

True or false: A patient with an aortic valve replacement is taking warfarin (Coumadin); an INR of 2.5 is therapeutic.

True

CHF Class 4

Unable to carry out any physical activity without discomfort. Symptoms at rest.

Side effects

Unintended effect occurring at normal dose.

Vancomycin

Vancocin Vancoled Therapeutic use: Active only against gram + bacteria Staph aureus and staphylococcus epidermiditis (Mrsa) Reserved for serious infection by MRSA - most strains are still sensitive to this drug Adverse effects: Ototoxicity Red man syndrome Due to release of histamine-(infuse too quickly--face, neck, chest wall becomes pink)

Epinephrine

Vasoconstrictor

Migraine

Vasodialation of brain blood vessels

A patient wants to know when they should take their warfarin (Coumadin). What is your response to this question?

Warfarin should be taken in the AM because INR laboratory tests are done in the AM because that is when the blood level will be the lowest. So basically taking warfarin (Coumadin) in the AM permits more accurate INR results

What is the range for potassium: ___ mEq/L to ___ mEq/L. Why is potassium level important when administering digoxin?

What is the range for potassium: __3.5___ mEq/L to __5___ mEq/L. Why is potassium level important when administering digoxin? It is important to measure because when the levels are too high or too low the patient's cardiac function may be compromised. In fact, hypokalemia can potentiate (make worse) digoxin toxicity. Diuretics are commonly given in patients with heart failure (CHF) so this increases the risk even more.

Ezetimibe

Zetia Mechanism of action: Acts on brush border of small intestine Inhibits cholesterol absorption

Acyclovir

Zovirax Mechanism of action: Inhibits viral DNA polymerase Becomes incorporated into viral strand Therapeutic use: HSV - 1: eye, mouth, lips HSV - 2: genital Varicella - zoster (VZV) chicken pox and shingles (reactivation) and post herpetic neuralgia

__ __ is an index of average glucose levels over the prior 2 to 3 months.

__Hemoglobin A1c __ is an index of average glucose levels over the prior 2 to 3 months.

Gout

accumulation of uric acid crystals in joint Caused by either Decreased excretion of uric acid Increased metabolism of nucleic acids

Calcium channel blockers: dihydropyridines

amlodipine, nifedepine Mechanism of action: selective types of CCB primarily act on the arterioles However they activate the baroreceptor reflex Indications: Angina, hypertension Adverse effects: reflex tachycardia Indications: angina, HTN Adverse effects: Reflex tachycardia Dizziness Lightheadedness Drug interactions: Digoxin - increased level to 60%, AV conduction Beta blockers - counteract the reflex tachycardia

COX 2 (Cyclooxygenase-2)

an enzyme that acts to speed up the production of certain chemical messengers, called prostaglandins that play a key role in in promoting inflammation. When cox-2 activity is blocked, inflammation is reduced. Unlike cox-1, cox-2 is active only at the site of inflammation, not in the stomach.

COX I (Cyclooxygenase-1)

an enzyme that acts to speed up the production of certain chemical messengers, called prostaglandins, in a variety of areas of the body such as the stomach, kidneys, and sites of inflammation.

Cephalosporins

cefoxitin cefotaxime cephalexin cefepime Mechanism of action: Bind to penicillin binding proteins / disrupt cell wall Resistance is main problem

Drug schedule 5

cough preparations

Drug schedule 4

diazepam, alprazolam, volume

COMT Inhibitors

entacapone (Comtan) tolcapone (Tasmar) Mechanism of action: Inhibit metabolism of levodopa in the peripheral tissues and intestine Used in conjunction to prolong its half life Nursing care: Evaluate response - improvement in motor ability Entacapone (Comtan) - assess for hyperactivity, hallucinations, or uncontrollable movements of tongue, lips, or face tolcapone (Tasmar) - assess for signs of liver damage

Loop Diuretics

furosemide (Lasix) bumetanide (Bumex) Mechanism of action: Produce the largest amount of diuresis Acts in thick segment of the ascending limb of Henle's Loop Kinetics: Administered orally IV Indications: Pulmonary edema with CHF Edema (hepatic, cardiac or renal origin) That has not been responsive with less strong CHF Cirrhosis Renal failure Drug interactions: Digoxin Ototoxic drugs K sparing diuretics Lithium Antihypertensive agents NSAIDs

Fibric acid derivatives

gemfibrozil (Lopid), fenofibrate (Tricor) Mechanism of Action: Lower triglycerides, Raises HDL

Sulfonylureas

glipizide (Glucotrol, Glucotrol XL), Glyburide (Micronase), glimepiride (Amaryl) Derivatives of sulfa antibiotics Mechanism of action: Stimulate release of pancreatic islet cells Must have some beta cell function left Therapeutic use: Only Type II diabetes

Anticholinergic / Antihistamines

hydroxyzine (Vistaril) - Scopolamine - often used "sea sickness" Meclizine (Antivert) - often used vertigo Side effects: Dry mouth Drowsiness Blurred vision

Calcium level

level 9 - 10 mg / dL

Potassium leves

levels 3.5 mEq - 5 mEq PO: irritating to stomach IV: dangerous, administer slowly diluted 10 - 25 mEq/hour (hospital policy)

Interferon alfa - 2a Interferons

naturally occurring proteins. Roferon Mechanism of action: Bind to specific receptors on cancer cell Adverse effect: Interferon may cause mental slowing, confusion, and lethargy

Vincristine Natural product

oncovin Mechanism of action: Cell cycle specific, M phase Blocks mitosis Peripheral neuropathy, hypersensitivity Paclitaxel (Taxol) - G2 phase Action: Inhibits cell division Severe hypersensitivity reactions Given slowly and with steroids by IV push

Phenothiazines

prochlorperazine (Compazine) metoclopramide (Reglan) Mechanism of action: Blocks dopamine receptors in CTZ Adverse effects: extrapyramidal movements Dose related anticholingeric effects Neuroleptic malignant syndrome Agranulocytosis

Prostogladinds

promote the production of a protective natural mucus lining. They also interact within certain cells that are responsible for inflammation and other functions.

Tetracyclines

sumycin minocycline minocin Mechanism of action: Drugs bind to the 30S ribosomal subunit - bind transfer RNA to the messenger Drug Interactions: Metal ions - Ca, Fe, Mg, Al Decreases absorption of antibiotic Adverse effects: GI irritation Suprainfection Liver toxicity Renal toxicity: exacerbate renal disease with preexisting disease Increase blood pressure uncontrolled diabetes Photosensitivity - dizzy, lightheadedness

Agonist

to enhance or mimic

Pharmacodynamic interaction

two or more drugs compete for same site.

Warfrain food interactions

vitamin k

What teaching should the nurse include when instructing a patient about oral nitrates?

• How to take their medication • Maintaining safety while taking the medications • Monitoring effects of the antianginal medications It is also important to instruct the patient on when to seek emergency medical services.

What teaching should the nurse include when instructing a patient about oral nitrates?

• How to take their medication • Maintaining safety while taking the medications • Monitoring effects of the antianginal medications • It is also important to instruct the patient on when to seek emergency medical services.

. Which situations may result in decrease needs for insulin?

• Hypothyroid • Alcohol • Exercise

. what dietary recommendations can the nurse make which will make medications for lipedemia work more effectively?

• Increase dietary supplementation of soluble fiber (psyllium, oat bran, fruits and vegetables • Dietary supplementation with cholesterol lowering margarines • Emphasize regular aerobic exercise (usually 30 minutes at least 3x a week) • Start a weight loss program if the patient is overweight; weight loss can increase HDL and decrease LDL • Assist patient to develop a smoking cessation plan; HDL levels are higher in nonsmokers • If patient has elevated serum triglycerides, ingest low amounts of saturated fat and cholesterol

Which conditions in a hospitalized patient would result in a patient needing higher doses of insulin due to hyperglycemia.:

• Stress • Infection • Epinephrine (potent vasoconstrictor with catecholamine effects - increases glucose). • NSAIDs • Diuretics • Steroids • Thyroid


Related study sets

chapter 6 growth in length of long bones

View Set

Financial Accounting Chapter 2 Key Concepts

View Set

COM292 Organizational Com. Exam 1

View Set

Physics II: Motion, Force, Energy and Equilibrium

View Set

Mang 4468-Ch5 Job-based Structures & Job Evaluations

View Set